Once we can see them, it’s too late

[updates: here’s the paper, and here’s Robin’s brief response to some of the comments here]

This month Robin Hanson, the famous and controversy-prone George Mason University economics professor who I’ve known since 2004, was visiting economists here in Austin for a few weeks. So, while my fear of covid considerably exceeds Robin’s, I met with him a few times in the mild Texas winter in an outdoor, socially-distanced way. It took only a few minutes for me to remember why I enjoy talking to Robin so much.

See, while I’d been moping around depressed about covid, the vaccine rollout, the insurrection, my inability to focus on work, and a dozen other things, Robin was bubbling with excitement about a brand-new mathematical model he was working on to understand the growth of civilizations across the universe—a model that, Robin said, explained lots of cosmic mysteries in one fell swoop and also made striking predictions. My cloth facemask was, I confess, unable to protect me from Robin’s infectious enthusiasm.

As I listened, I went through the classic stages of reaction to a new Hansonian proposal: first, bemusement over the sheer weirdness of what I was being asked to entertain, as well as Robin’s failure to acknowledge that weirdness in any way whatsoever; then, confusion about the unstated steps in his radically-condensed logic; next, the raising by me of numerous objections (each of which, it turned out, Robin had already thought through at length); finally, the feeling that I must have seen it this way all along, because isn’t it kind of obvious?

Robin has been explaining his model in a sequence of Overcoming Bias posts, and will apparently have a paper out about the model soon the paper is here! In this post, I’d like to offer my own take on what Robin taught me. Blame for anything I mangle lies with me alone.

To cut to the chase, Robin is trying to explain the famous Fermi Paradox: why, after 60+ years of looking, and despite the periodic excitement around Tabby’s star and ‘Oumuamua and the like, have we not seen a single undisputed sign of an extraterrestrial civilization? Why all this nothing, even though the observable universe is vast, even though (as we now know) organic molecules and planets in Goldilocks zones are everywhere, and even though there have been billions of years for aliens someplace to get a technological head start on us, expanding across a galaxy to the point where they’re easily seen?

Traditional answers to this mystery include: maybe the extraterrestrials quickly annihilate themselves in nuclear wars or environmental cataclysms, just like we soon will; maybe the extraterrestrials don’t want to be found (whether out of self-defense or a cosmic Prime Directive); maybe they spend all their time playing video games. Crucially, though, all answers of that sort founder against the realization that, given a million alien civilizations, each perhaps more different from the others than kangaroos are from squid, it would only take one, spreading across a billion light-years and transforming everything to its liking, for us to have noticed it.

Robin’s answer to the puzzle is as simple as it is terrifying. Such civilizations might well exist, he says, but if so, by the time we noticed one, it would already be nearly too late. Robin proposes, plausibly I think, that if you give a technological civilization 10 million or so years—i.e., an eyeblink on cosmological timescales—then either

  1. the civilization wipes itself out, or else
  2. it reaches some relatively quiet steady state, or else
  3. if it’s serious about spreading widely, then it “maxes out” the technology with which to do so, approaching the limits set by physical law.

In cases 1 or 2, the civilization will of course be hard for us to detect, unless it happens to be close by. But what about case 3? There, Robin says, the “civilization” should look from the outside like a sphere expanding at nearly the speed of light, transforming everything in its path.

Now think about it: when could we, on earth, detect such a sphere with our telescopes? Only when the sphere’s thin outer shell had reached the earth—perhaps carrying radio signals from the extraterrestrials’ early history, before their rapid expansion started. By that point, though, the expanding sphere itself would be nearly upon us!

What would happen to us once we were inside the sphere? Who knows? The expanding civilization might obliterate us, it might preserve us as zoo animals, it might merge us into its hive-mind, it might do something else that we can’t imagine, but in any case, detecting the civilization would presumably no longer be the relevant concern!

(Of course, one could also wonder what happens when two of these spheres collide: do they fight it out? do they reach some agreement? do they merge? Whatever the answer, though, it doesn’t matter for Robin’s argument.)

On the view described, there’s only a tiny cosmic window in which a SETI program could be expected to succeed: namely, when the thin surface of the first of these expanding bubbles has just hit us, and when that surface hasn’t yet passed us by. So, given our “selection bias”—meaning, the fact that we apparently haven’t yet been swallowed up by one of the bubbles—it’s no surprise if we don’t right now happen to find ourselves in the tiny detection window!

This basic proposal, it turns out, is not original to Robin. Indeed, an Overcoming Bias reader named Daniel X. Varga pointed out to Robin that he (Daniel) shared the same idea right here—in a Shtetl-Optimized comment thread—back in 2008! I must have read Daniel Varga’s comment then, but (embarrassingly) it didn’t make enough of an impression for me to have remembered it. I probably thought the same as you probably thought while reading this post:

“Sure, whatever. This is an amusing speculation that could make for a fun science-fiction story. Alas, like with virtually every story about extraterrestrials, there’s no good reason to favor this over a hundred other stories that a fertile imagination could just as easily spin. Who the hell knows?”

This is where Robin claims to take things further. Robin would say that he takes them further by developing a mathematical model, and fitting the parameters of the model to the known facts of cosmic history. Read Overcoming Bias, or Robin’s forthcoming paper, if you want to know the details of his model. Personally, I confess I’m less interested in those details than I am in the qualitative points, which (unless I’m mistaken) are easy enough to explain in words.

The key realization is this: when we contemplate the Fermi Paradox, we know more than the mere fact that we look and look and we don’t see any aliens. There are other relevant data points to fit, having to do with the one sample of a technological civilization that we do have.

For starters, there’s the fact that life on earth has been evolving for at least ~3.5 billion years—for most of the time the earth has existed—but life has a mere billion more years to go, until the expanding sun boils away the oceans and makes the earth barely habitable. In other words, at least on this planet, we’re already relatively close to the end. Why should that be?

It’s an excellent fit, Robin says, to a model wherein there are a few incredibly difficult, improbable steps along the way to a technological civilization like ours—steps that might include the origin of life, of multicellular life, of consciousness, of language, of something else—and wherein, having achieved some step, evolution basically just does a random search until it either stumbles onto the next step or else runs out of time.

Of course, given that we’re here to talk about it, we necessarily find ourselves on a planet where all the steps necessary for blog-capable life happen to have succeeded. There might be vastly more planets where evolution got stuck on some earlier step.

But here’s the interesting part: conditioned on all the steps having succeeded, we should find ourselves near the end of the useful lifetime of our planet’s star—simply because the more time is available on a given planet, the better the odds there. I.e., look around the universe and you should find that, on most of the planets where evolution achieves all the steps, it nearly runs out the planet’s clock in doing so. Also, as we look back, we should find the hard steps roughly evenly spaced out, with each one having taken a good fraction of the whole available time. All this is an excellent match for what we see.

OK, but it leads to a second puzzle. Life on earth is at least ~3.5 billion years old, while the observable universe is ~13.7 billion years old. Forget for a moment about the oft-stressed enormity of these two timescales and concentrate on their ratio, which is merely ~4. Life on earth stretches a full quarter of the way back in time to the Big Bang. Even as an adolescent, I remember finding that striking, and not at all what I would’ve guessed a priori. It seemed like obviously a clue to something, if I could only figure out what.

The puzzle is compounded once you realize that, even though the sun will boil the oceans in a billion years (and then die in a few billion more), other stars, primarily dwarf stars, will continue shining brightly for trillions more years. Granted, the dwarf stars don’t seem quite as hospitable to life as sun-like stars, but they do seem somewhat hospitable, and there will be lots of them—indeed, more than of sun-like stars. And they’ll last orders of magnitude longer.

To sum up, our temporal position relative to the lifetime of the sun makes it look as though life on earth was just a lucky draw from a gigantic cosmic Poisson process. By contrast, our position relative to the lifetime of all the stars makes it look as though we arrived crazily, freakishly early—not at all what you’d expect under a random model. So what gives?

Robin contends that all of these facts are explained under his bubble scenario. If we’re to have an experience remotely like the human one, he says, then we have to be relatively close to the beginning of time—since hundreds of billions of years from now, the universe will likely be dominated by near-light-speed expanding spheres of intelligence, and a little upstart civilization like ours would no longer stand a chance. I.e., even though our existence is down to some lucky accidents, and even though those same accidents probably recur throughout the cosmos, we shouldn’t yet see any of the other accidents, since if we did see them, it would already be nearly too late for us.

Robin admits that his account leaves a huge question open: namely, why should our experience have been a “merely human,” “pre-bubble” experience at all? If you buy that these expanding bubbles are coming, it seems likely that there will be trillions of times more sentient experiences inside them than outside. So experiences like ours would be rare and anomalous—like finding yourself at the dawn of human history, with Hammurabi et al., and realizing that almost every interesting thing that will ever happen is still to the future. So Robin simply takes as a brute fact that our experience is “earth-like” or “human-like”; he then tries to explain the other observations from that starting point.

Notice that, in Robin’s scenario, the present epoch of the universe is extremely special: it’s when civilizations are just forming, when perhaps a few of them will achieve technological liftoff, but before one or more of the civilizations has remade the whole of creation for its own purposes. Now is the time when the early intelligent beings like us can still look out and see quadrillions of stars shining to no apparent purpose, just wasting all that nuclear fuel in a near-empty cosmos, waiting for someone to come along and put the energy to good use. In that respect, we’re sort of like the Maoris having just landed in New Zealand, or Bill Gates surveying the microcomputer software industry in 1975. We’re ridiculously lucky. The situation is way out of equilibrium. The golden opportunity in front of us can’t possibly last forever.

If we accept the above, then a major question I had was the role of cosmology. In 1998, astronomers discovered that the present cosmological epoch is special for a completely different reason than the one Robin talks about. Namely, right now is when matter and dark energy contribute roughly similarly to the universe’s energy budget, with ~30% the former and ~70% the latter. Billions of years hence, the universe will become more and more dominated by dark energy. Our observable region will get sparser and sparser, as the dark energy pushes the galaxies further and further away from each other and from us, with more and more galaxies receding past the horizon where we could receive signals from them at the speed of light. (Which means, in particular, that if you want to visit a galaxy a few billion light-years from here, you’d better start out while you still can!)

So here’s my question: is it just a coincidence that the time—right now—when the universe is “there for the taking,” potentially poised between competing spacefaring civilizations, is also the time when it’s poised between matter and dark energy? Note that, in 2007, Bousso et al. tried to give a sophisticated anthropic argument for the value of the cosmological constant Λ, which measures the density of dark energy, and hence the eventual size of the observable universe. See here for my blog post on what they did (“The array size of the universe”). Long story short, for reasons that I explain in the post, it turns out to be essential to their anthropic explanation for Λ that civilizations flourish only (or mainly) in the present epoch, rather than trillions of years in the future. If we had to count civilizations that far into the future, then the calculations would favor values of Λ much smaller than what we actually observe. This, of course, seems to dovetail nicely with Robin’s account.

Let me end with some “practical” consequences of Robin’s scenario, supposing as usual that we take it seriously. The most immediate consequence is that the prospects for SETI are dimmer than you might’ve thought before you’d internalized all this. (Even after having interalized it, I’d still like at least an order of magnitude more resources devoted to SETI than what our civilization currently spares. Robin’s assumptions might be wrong!)

But a second consequence is that, if we want human-originated sentience to spread across the universe, then the sooner we get started the better! Just like Bill Gates in 1975, we should expect that there will soon be competitors out there. Indeed, there are likely competitors out there “already” (where “already” means, let’s say, in the rest frame of the cosmic microwave background)—it’s just that the light from them hasn’t yet reached us. So if we want to determine our own cosmic destiny, rather than having post-singularity extraterrestrials determine it for us, then it’s way past time to get our act together as a species. We might have only a few hundred million more years to do so.

Update: For more discussion of this post, see the SSC Reddit thread. I especially liked a beautiful comment by “Njordsier,” which fills in some important context for the arguments in this post:

Suppose you’re an alien anthropologist that sent a probe to Earth a million years ago, and that probe can send back one high-resolution image of the Earth every hundred years. You’d barely notice humans at first, though they’re there. Then, circa 10,000 years ago (99% of the way into the stream) you begin to see plots of land turned into farms. Houses, then cities, first in a few isolated places in river valleys, then exploding across five or six continents. Walls, roads, aqueducts, castles, fortresses. Four frames before the end of the stream, the collapse of the population on two of the continents as invaders from another continent bring disease. At T-minus three frames, a sudden appearance of farmland and cities on the coasts those continents. At T-minus two frames, half the continent. At the second to last frame, a roaring interconnected network of roads, cities, farms, including skyscrapers in the cities that were just trying villas three frames ago. And in the last frame, nearly 80 percent of all wilderness converted to some kind of artifice, and the sky is streaked with the trails of flying machines all over the world.

Civilizations rose and fell, cultures evolved and clashed, and great and terrible men and women performed awesome deeds. But what the alien anthropologist sees is a consistent, rapid, exponential explosion of a species bulldozing everything in its path.

That’s what we’re doing when we talk about the far future, or about hypothetical expansionist aliens, on long time scales. We’re zooming out past the level where you can reason about individuals or cultures, but see the strokes of much longer patterns that emerge from that messy, beautiful chaos that is civilization.

Update (Jan. 31): Reading the reactions here, on Hacker News, and elsewhere underscored for me that a lot of people get off Robin’s train well before it’s even left the station. Such people think of extraterrestrial civilizations as things that you either find or, if you haven’t found one, you just speculate or invent stories about. They’re not even in the category of things that you have any serious hope to reason about. For myself, I’d simply observe that trying to reason about matters far beyond current human experience, based on the microscopic shreds of fact available to us (e.g., about the earth’s spatial and temporal position within the universe), has led to some of our species’ embarrassing failures but also to some of its greatest triumphs. Since even the failures tend to be relatively cheap, I feel like we ought to be “venture capitalists” about such efforts to reason beyond our station, encouraging them collegially and mocking them only gently.

265 Responses to “Once we can see them, it’s too late”

  1. Margaret Says:

    Robin’s “new” idea is already well explored in A Fire Upon the Deep, albeit a fictional novel.

  2. D. L. Yonge-Mallo Says:

    Isn’t this just a variant of the “Dark Forest theory” solution to the Fermi paradox?

  3. Guan Says:

    1.Have you slept last night ?
    2.Have you talked with Dana?
    3. Each country has a president and a government caring about the fate of humanity, whether they are responsible/qualified or not.
    4. You can not help anyone unless you seek help and take care of yourself first.
    5. Thanks for the new post, always full of surprises and excitement even at you bluer moment.But I knew very little about it very new for me
    6. Regular physical exercises will boost your dopamine level and immune system, buy something like a bicycle and USE it at least 400kcal daily if you haven’t.
    7 carefully choose and make an appointment with a phychologist if you still not well for after seeking friends’ help.

  4. Justin Says:

    ‘By contrast, our position relative to the lifetime of all the stars makes it look as though we arrived crazily, freakishly early—not at all what you’d expect under a random model”

    If you take sun-earth entropy production into account and compare it to the total entropy of life on earth today you could say that we’ve arrived crazily and freakishly late. A powerful designer could have done it in about 4 months if you take an approximate upper bound (so even 6 days wouldn’t be inconceivable). The fact that it has taken so long can only mean it is a random process surely?

    PS thank you Scott for all your great work and lecture notes etc. Without it I’d have nothing better to do.

  5. franzr Says:

    To me, another one of those “big if true” concepts…unlike, say, Sim hypothesis or Boltzmann Brains, I find this idea actually very inspiring. (Like, with the “Avengers’ Endgame” theme playing in the back of my mind). A reminder that the universe is not only ours for the taking but that there’s a soft deadline in the near(ish, by cosmological timescales in the one hand, but also concretely by AI development timelines) future to do so.

  6. Unicephalon Says:

    > So, given our “selection bias”—meaning, the fact that we haven’t yet been swallowed up by one of the bubbles…

    How would we know if we have? Hanson — who would have made an excellent theologist, of the “how many angels can dance on the head of a pin” school — is assuming the existence of Godlike beings, and if Godlike beings who move at the speed of light want to place us in a zoo or planetarium, that’s exactly what they’ll do.

    I realize that the argument above invokes a Cartesian demon, but it’s ultimately no more or less intractible than Hanson’s own position.


    >Now is the time when the early intelligent beings like us can still look out and see quadrillions of stars shining to no apparent purpose, just wasting all that nuclear fuel in a near-empty cosmos, waiting for someone to come along and put the energy to good use.
    > […]
    > The key realization is this: when we contemplate the Fermi Paradox, we know more than the mere fact that we look and look and we don’t see any aliens.

    To date, we’ve looked almost exclusively for radio signals and Dyson shperes. It’s extremely easy to explain the lack of both.

    The math for fusion, if you look into it, enables efficient reactions that can produce output equivalent to the sun in a vastly smaller area. (e.g., a spherical reactor with a 50km radius.) It’s foolish to assume that a superintelligent species will encircle an entire star when they can simply build a better, smaller, more stable engine.

    As long-term fuel sources go, white dwarfs are near-optimal, and the fate of most stars is to end up as a stable white dwarf. Black holes are, in theory, also extremely efficient. Big, fat, unstable monsters like Betelgeuse and Aldebaran are the worst possible fuel sources. It should be no surprise that they’re allowed to shine — now and in the future — on this account. (Though I suppose a Godlike civilization may attempt to accelerate their end.)

    ‘Oumuamua may well be technological in origin. As celestial bodies go, it’s an outlier’s outlier. I believe that a truly compelling natural origin has yet to be hypothesized.

  7. Tez Says:

    >if we want human-originated sentience to spread across the universe, then the sooner we get started the better!

    The english seeded their empire with cricketers, and then were very distressed when they then found themselves losing to their creation. They also learned there is no guarantee you will maintain cordial relations with even closely related humans if you are separated from them for only a little while!

  8. LGS Says:

    The diameter of the Milky Way is only ~100k lightyears. You still get a Fermi paradox for the Milky Way; 10 million years is not an “eyeblink” on the timescale of the tiny Milky Way. In fact, the diameter of the local group is merely ~10 million lightyears.

    Was SETI ever even looking for extraterrestrial life outside the local group? I always thought people were looking for aliens *in our galaxy*, not in a different galaxy cluster (the closest one is ~300 million lightyears away…) And same for the Fermi paradox: I always thought it was referring to the Milky Way; the explanation for why we don’t hear from aliens in other galaxy clusters might simply be that it’s very hard to hear things from 300 million lightyears away.

    I do agree that Hanson’s proposal has an appeal in explaining why our experience is so cosmologically early (only 13.7 billion years after the big bang). I just feel like this is more-or-less the only thing it explains. If someone was bothered by how there is no alien contact from outside our local group, but *not* bothered by how there is no alien contact from inside our local group, that someone could always explain it by positing that it’s hard to communicate across a 300 million lightyear chasm. Hanson’s proposal doesn’t add much to this.

  9. Vampyricon Says:

    UNIVERSAL DOMINATION! *laughs evilly*

  10. rsegal Says:

    Why should we expect to see the expanding sphere coming? Surely aliens wouldn’t roll around shining as bright as stars, and there’s a whole lot of space to dump their photons into.

    We have lost and found probes within our own solar system – and we knew where they were supposed to be! Would we even be able to find the Voyager probes if we didn’t know where to look?

  11. Gerard Says:

    Scott:

    > So Robin simply accepts as a brute fact that our experience is “earth-like” or “human-like”; he then tries to explain the other observations from that starting point.

    That seems to me like a very flawed assumption. How do you even define “us” so that “our” experience is constrained to lie in some small subset of all possible experiences ?

    The only way I can see this making sense is if advanced civilizations inevitably eliminate sentience.

  12. Scott Says:

    Margaret #1:

      Robin’s “new” idea is already well explored in A Fire Upon the Deep, albeit a fictional novel.

    I’ve read Fire Upon the Deep, and the ideas in it (centered around “Zones of Thought”) strike me as almost totally unrelated to what we’re talking about here.

  13. Scott Says:

    D. L. Yonge-Mallo #2:

      Isn’t this just a variant of the “Dark Forest theory” solution to the Fermi paradox?

    I wouldn’t call it that. It’s a dangerous forest, to be sure, but the potential adversaries are easily seen once they arrive—it’s just that they can only travel at the speed of light. So the civilizations are not hiding from each other; they’re just preparing as best they can for each others’ inevitable arrival!

  14. Jon Says:

    Even accepting the 1,2,3 trichotomy, isn’t it the relative frequency of cases 2 and 3 and how this compares to their signal output that is relevant?

  15. Adam B Says:

    Can a layperson achieve confidence in the estimates of cosmology? I don’t know much hard science, but if particle physicists announced tomorrow that the proton was 1000x lighter or heavier than previously thought, I would be shocked to my core. But if cosmologists announced a 1000x correction in the age of the sun, the age of the universe, or the time left until the sun explodes, I would just shrug.

  16. Noah Lidell Says:

    I really like this concept! It feels simple and rings true in Bayes flavored way.

    One scary, albeit unlikely, possibility is that an expanding sphere of alien super intelligence enveloped Earth around, say, one hundred thousand years ago while human intelligence was in its near animal infancy (or even before this). At the time we would have lacked the technology to observe and record this event. We could now be in a “zoo” situation where an alien super intelligence has placed difficult to observe barriers upon our civilization that is limiting our technological development.

  17. Scott Says:

    Adam B #15:

      if particle physicists announced tomorrow that the proton was 1000x lighter or heavier than previously thought, I would be shocked to my core. But if cosmologists announced a 1000x correction in the age of the sun, the age of the universe, or the time left until the sun explodes, I would just shrug.

    I think that was a reasonable attitude before nuclear physics was understood (in the case of the sun) and before precision measurements of cosmological parameters (in the case of the universe), but it has not been reasonable for generations. If you trust the physicists about the mass of the proton—which you should! 🙂 —then on the same basis you should trust them about the approximate ages of the sun and of the Big Bang, allowing perhaps a 10% error (the values have actually been more stable than that for decades) but certainly not a 1000x (!) error.

  18. Taymon A. Beal Says:

    I’m curious how much pause it gives you that this argument hinges so heavily on the anthropic principle. I confess that I find it difficult to trust anthropic reasoning, given its tendency to result in apparent paradoxes (example).

  19. Edward Measure Says:

    I have no idea how original Hanson’s idea is, but it is interesting in that some key underpinnings are likely to be tested in the next few decades (assuming humans manage to survive that long.) In particular we should get a lot more clarity on the question of how many planets are suitable for life and possibly even on whether some of them seem to support life. We might also figure out how life originated on Earth. The harder question of how difficult it is for space travelling technological life to develop may depend on SETI making a discovery.

  20. Joshua Zelinsky Says:

    As noted, variants of this idea have been discussed and proposed elsewhere. I’m generally skeptical of the idea (although I find it very intriguing) since it requires that one can expand at close to light-speed. But if you are going to use the resources of the area you expanded into, you are going to need to slow down in a controlled fashion once you get there. And that aside from the whole going-at-90% of light speed and you run into a stray dust spect and you get a big boom. And if one assumes that one is going at anything below about 95% of light speed or so, then you will probably see them coming for a long time. But Robin’s version has the virtue of helping explain why we’re not around a red dwarf in the far future. I have not seen that point before, and his model is certainly interesting.

  21. Gerard Says:

    Taymon A. Beal #18

    I didn’t read the entire, very long, post you linked but from where it seems to be going I suspect that any paradoxes it arrives at are probably due to the assumption that consciousness is computable (and therefore copyable) rather than the anthropic principle.

  22. dankane Says:

    I feel like the “the sooner we get started the better” conclusion is exactly the opposite of the conclusion you should draw. We have a billion years to expand into space and we expect that most universe-tiling civilizations spend almost all of the lifetime of their star before they start. Furthermore, on the timescales we are operating at these days a billion years is practically forever. The human population has been doubling at a rate of better than once a century lately. I calculated a bit ago that at around a quadrillion people, we overheat the Earth just from our metabolic processes. At the current rates we should be there in another couple of millennia, much much less than the billion years we have left. Compared to the timescales that current human civilization is operating on, we have practically forever to get to space.

    What we should learn from the Fermi paradox is that it is very very rare to become a universe-tiling civilization. If there is a great filter, it might be behind us already, but there’s a reasonable chance that it is still ahead of us. We should be *extra* careful about possible civilization destroying threats like global warming, nuclear war, or global pandemics. These seem much more likely to prevent us from reaching the status of an expanding light cone than not acting quickly enough will.

    If we focus too much on being fast, we may forget to pay attention to our immediate problems.

  23. Sam Harsimony Says:

    What I am confused about is the assumption that alien civilizations will selfishly expand: this is an equilibrium for simple strategic agents, but civilizations have the opportunity to reflect on this fact.

    A civilization seeing an empty galaxy but with little desire to expand would know that eventually the galaxy will be taken by a more selfish species. Reflecting on this, they might choose to expand (despite their preference not to expand) in order to ensure that the galaxy is shared equally between different civilizations (or enforce whatever version of morality they see fit). Their expansion is limited by how soon the next expansionary civilization will arrive, which means that they could move much slower and hide their presence from fledgling civilizations.

    This would explain the observations and the apparent improbability of humanity being the first sophisticated civilization.

    Under this scenario, either humanity is this first civilization and must decide how to deal with other life in the universe, or humanity is being allowed to grow by a hidden species.

    Note: before claiming that sub-components of this civilization will themselves become expansionist, remember that the first civilization has time to build systems that ensure that this doesn’t happen in any reasonable time-frame.

  24. Oleg S. Says:

    It does not follow from the assumption #3 that civilization should expand at a speed of light. I think there are several arguments against rapid expansion:

    1. Imagine it takes you 1 year to build a von Neumann probe, and the probe itself is capable of reaching 99% of c. You can then send the probes to whatever star system you want, but the density of these arrived probes will diminish as square distance from the home system. So it would look like an expanding sphere of smaller and smaller density untill the distance between two nearest 1st generation ships is too huge for them to see each other. The actual numbers will depend on the production capacity of the home planet, but the logic remains.

    2. The expanding sphere however will be seen, but the main contributor for the expansion will be the starships of N-th generation, descended from the original probes. The actual rate of expansion as seen from the outside observer will be mainly controlled by the reproduction rate of the probes, and will be much lower than the max speed of each probe.

    3. Another tradeoff stems from the fact that there is little point in manufacturing von Neumann probes which are too unlike to your own civilization. These will exhaust the resources of the nearby stars, but won’t help your civilization spread. So, you may want invest into more slowly reproducing probes which preserve most of your civilization’ values rather than into rapidly reproducing blights. Occasional spillovers might occur, but they will have a different signature than the more slow probes from mature civilizations.

  25. dankane Says:

    I also think Robin missed some possibilities in his classification:
    4. A nomadic civilization that grows relatively slowly but moves quickly in order to cover a lot of ground. This would be a natural consequence if you wanted to accrue resources quickly, but wanted your civilization to maintain not too long light speed communication from one end to the other.
    5. A civilization that expands *only* for defensive purposes as suggested by Sam#23 might reasonably stop expanding once it has colonized a single galaxy. I guess this might technically fall under case 1, but unlike the typical case 1 would probably be easy to detect even from far away.

  26. Robert Says:

    One thing strikes me right off:

    1. the civilization wipes itself out, or else
    2. it reaches some relatively quiet steady state, or else

    With all the number of civilizations proposed going through this we should see evidence in quite a few places of the “lead up” to those outcomes before the signals stop. Yes, they might stop at the endpoints but the “early days” should provide quite a few artifacts to be seen unless the number of civilizations (for some reason) is much less than proposed.

  27. Will P Says:

    I don’t understand this point: “By that point, though, the expanding sphere itself would be nearly upon us!” does “nearly” mean on the order of a million years? then perhaps it is not so terrifying – we are much more likely to reach scenario 1 before then (and unlikely to end up as zoo animals)

  28. 1Zer0 Says:

    That near speed of light “sphere” able to transform star systems seems incredibly optimistic.
    If the best spaceship possible build by a civilization that reached a technological singularity, where not further improvement of tech is achievable due to restrictions in the laws of physics of our observable universe, would fail after 200 years of space travel in average, then they likely wont get very far with their exploration. Even if some of those spaceships have some sort of self repair ability, they would eventually run out of raw resources and be forced to land to mine new resources – if possible at all. Within the ship they have to carry all the resource to bootstrap, such that when they land they can either repair their spaceship or found a colony to start processing more materials and eventually build a new spaceship. This likely requires a large passenger crew which in turns requires a large spaceship.
    Let’s take earth as an example. Surely we can reach Mars, that’s “easy”. But what about the next star? Is there any possible way a technological device like a spaceship (or a fleet thereof) which supports life can last the journey to the next star together with its passengers + the passengers being able to bootstrap a colony with all the industry needed to maintain their spaceship?
    I think there may very well other life forms, possible intelligent, in our observable universe.
    The Cosmos however is not suitable for space travel of sentient lifeforms. Certainly there exists something like this, “somewhere else” with physical laws better qualified to support such travels. Humanity though is going to perish in the solar system.

  29. fred Says:

    “if we want human-originated sentience to spread across the universe”

    Do we actually really want that?
    Just because we were born as members of the human team, then we have to spread ourselves like a virus/cancer running amok?
    Any sort of human tribe can actually claim the same thing (the Nazis, the communists, ancient imperial Rome,…).

    I think this inability to question the place of our own tribe in the grand scheme of things is actually the root of why we’ll never survive.
    Human kind is lacking a necessary wisdom, and it’s too late to evolve it… but maybe our own machines will help us?

    Dolphins seem very smart and happy… but do they have a secret agenda (either explicit or built-in) to take over the earth then the universe? Maybe if not now, maybe if we give them enough time to evolve? Or maybe they’ve reached a steady-state equilibrium with their environment (at least until humans showed up).

    So the irony is that the key to surviving could require breaking away with that need for our own tribe to always control and win… but wouldn’t that make us weak if/when we run into aggressive alien civilizations?

    But we don’t need to turn to the stars to probe these questions, we see the very same dynamics in the current political life in the US!

  30. Ben Tilly Says:

    This is a variation on an argument due to Frank Tipler in 1981. Namely that once a Von Neumann probe is sent out, the ratio between how long it takes to evolve to the point of doing that the first time to how long it takes to have colonized the entire galaxy is such that it is unlikely that any other civilization in our galaxy would be in that window in the same time frame that we are arriving at that capability.

    Of the two arguments, I strongly prefer Tipler’s. Because Tipler’s argument does not presuppose that there is a technology that can continuously expand at close to the speed of light, instantly absorbing and taking advantage of any stars and planets in its way. Instead his argument works at much slower speeds that we know are achievable in principle with technologies that we can already imagine.

    True, it is hard to conceive of what limits might be surpassed by a sufficiently advanced intelligence. But basic principles of entropy should put some sort of limit on how fast a probe landing on an uncolonized planet can turn it into something that can contribute to the expansion. And furthermore the path of expansion from A to distant point B won’t be a straight line since each individual launch has to go in a zig-zag line. Either way its rate of expansion will be less than C. Which means that there will remain a gap between when they are detectable, and when they arrive. (Probably not a long-enough gap for a technologically backwards culture to prepare, but still a gap.)

  31. Unicephalon Says:

    Re: Scott #12
    Be that as it may, it is exactly the plot to David Zindell’s extremely good Neverness.
    In the book, “the Vild” is a mysterious and impenetrable region of space expanding at almost the speed of light, wherein stars are extinguished. I’d say more, but between that sentence and Hanson’s theory, if I say anything further I might ruin one of the book’s surprises!

  32. peter Says:

    I’ve been thinking on the fermi paradox a lot the last month or so. Something I haven’t run into in the 2 or 3 posts of Hanson’s I’ve just read are
    1) while earth is on the smaller end of radii of inner planets, it’s also on the larger end of planets where meaningful chemical orbital rockets can exist[1]. If a planet were much bigger than earth than “simple” chemical rocketry wouldn’t be sufficient to get into orbit, but still sufficient for ballistic nuclear war heads. Such a civilization would have a much higher barrier to entry to orbit with the likes of a lofstrom launch loop or a light-reflector, all the while the specter of nuclear war would be just as present.
    2) While stellar scale engineering projects should stand out pretty obviously, interstellar communications might be nearly undetectable compared with intrastellar communications. Using stellar bodies as transmission and reception foci it might mean simple kilo to mega watt power transmissions to cross entire galaxies [2]. Given these stations would be >= 500 au away from the sun, we could have several of these nodes in our very own solar system and they’d be too dim to notice without incredibly dedicated efforts.

    [1]https://www.realclearscience.com/blog/2017/07/06/if_earth_was_50_larger_we_might_be_stuck_here.html
    [2] https://www.centauri-dreams.org/2020/12/14/the-focal-radio-bridge/

  33. Gautam Goel Says:

    Hi Scott,

    The ideas discussed here, especially the idea of rapidly expanding spheres of civilizations consuming all resources in their path, were beautifully explored in Stephen Baxter’s sci-fi book, Manifold: Space (a spin-off of his earlier book, Manifold: Time, which is also excellent). In his book, alien intelligences are common; once they become sufficiently advanced, their civilizations tend to rapidly expand and consume all available resources, often to the detriment of other civilizations in their path. This pattern leads to some interesting phenomena: first, while the night sky might seem quiet at first, once we do encounter aliens, we tend to see their signals across many star systems in rapid succession. The reason is pretty obvious: there is only a brief period of time when we are on the surface of a sphere – a few years after our first observations of aliens, we are engulfed within their sphere and observe their signals from all over our stellar neighborhood. Another idea he plays with is the idea of “refugee” species, who attempt to flee oncoming spheres by evacuating ahead of their path instead of being consumed.

    Actually, he pushes this idea even further: in the book, our solar system was *already* engulfed in a few spheres millions of years ago. He suggests that this why Venus is such a hellscape: the aliens came, took the resources they wanted, and left behind a polluted mess. In the case of Venus, they left lots of greenhouse gases behind as the result of some chemical process used to extract resources; as a result, Venus quickly became the warmest planet in the solar system. It’s a fun twist on the Fermi paradox: signs of aliens are actually all around us, we are just too dumb to notice them.

    Another interesting idea he explores a bit is “ownership” of resources. Do the resource-rich asteroids in our solar system really belong to us? Or are they available to any alien race who happens to pass through? In the book, we first notice aliens by observing unexplainable infrared radiation from the asteroid belt (later revealed to be thermal emissions from their resource extraction). He suggests that these aliens will potentially crowd out humans; even if they are not overtly hostile, they could gobble up all the resources we would have used to expand our civilization.

    Highly recommend this book.

  34. Dániel Varga Says:

    Hey, that’s me, thanks for the shout-out, Scott! When I originally arrived at this idea, it was together with two related speculative ideas. I believe this is a perfect opportunity to mention those.

    Let’s say you want to execute some huge but highly parallelizable computation as fast as possible, say, within 10 million years. (It really is a huge computation.) Money is not a concern, neither is hundreds of galaxies completely destroyed in the process. How would you proceed? It seems like it would be useful if your computer expanded very fast, so that it could utilize resources encountered. A logical idea is a ball of Neumann probes expanding with speed (1-epsilon)c, and rearranging everything in their path into computer parts (and Neumann probes) that then use light to communicate with each other. How small can epsilon be, and how to choose epsilon if you want the fastest possible computer?

    In particular, is it worth the trouble to expand with _exactly_ the speed of light? (epsilon=0.) Obviously, only light is that fast, so that limits us to some very unusual computers. But there are exotic phenomena like photon-photon scattering, so it’s not clear to me that this can be ruled out a priori. (I am not a physicist.)

    And that, photon-photon scattering, leads me to a related question: disregarding computational speed for now, is it possible in principle to build a general purpose computer out of _nothing_ but photons in a vacuum? (Say, a mirror box with just vacuum and photons in it. Maybe not even the mirror box?)

    If my understanding is correct, it is possible in principle to build a general purpose computer out of nothing but idealized bouncing billiard balls in an empty box. To my layman’s eyes, the main theoretical difference between the bouncing billiard balls thought experiment and the scattering photons setup seem to be the extreme unreliability of photon-photon scattering. Is this some fundamental misunderstanding? If it’s not, can that unreliability be fixed with error correction? (Obviously still strictly remaining in the realm of thought experiments.)

  35. Bruce Smith Says:

    FYI, I also noticed these expanding civilizations would only become visible at the last moment, and that this (together with other ideas you point out) explained the Fermi paradox. I more or less tried to explain this to “everyone who would listen” (mainly at Foresight conferences in the 1990s, probably including Robin Hanson after he gave a related talk there), though I didn’t mathematically model it. (I didn’t believe this idea was likely to be original; I was influenced by a part of it Robin had already talked about back then, namely “a model wherein there are a few incredibly difficult, improbable steps along the way to a technological civilization like ours”.)

    The reference I can find most quickly dates from “a long time ago” relative to 2006 — see the start of this blog post by John Baez, its referenced diary entry, and my first comment on the post:

    https://johncarlosbaez.wordpress.com/2016/02/05/aggressively-expanding-civilizations/

    https://math.ucr.edu/home/baez/diary/october_2006.html#october27.06

    https://johncarlosbaez.wordpress.com/2016/02/05/aggressively-expanding-civilizations/#comment-77128

    That post has a lot of interesting related comments, by many people. Here are the most perhaps-original parts of my comment there:

    an expanding civilization might purposefully create lots of light in order to pre-accelerate surrounding matter into a more accessible frame of reference for eating it (as well as making it less dangerous to collide with);

    they might configure the surface and interior of their “bubble” as a large optical telescope (since a telescope is, after all, just a specialized quantum computer using internal interference of light to gain information about incoming light) which could resolve millimeter-sized features about 1000 times farther from them than their bubble’s radius; they have high motive to do so, in order to understand what they’re heading towards — particularly whether it might be about to turn into a competing bubble they might be “about to hit” on their 10^9-year planning timescale. (Since once it does, if it expands at 99.99% lightspeed, its “apparent motion” as they see it approaching will be 10000x faster than that.)

    that means that if you stand outside and look at a clear part of the sky (day or night), there’s a reasonable probability you’re visible to a far-away future intelligence. (Wave at them for me, or show them your interesting preprints. They can probably read your lips, having been analyzing everything they can see on Earth since they first noticed life there. Maybe they can even hear you, by seeing the optical wiggling of the background objects behind the sound waves you’re making in the air, or in some other way — I don’t know whether that’s possible. If it is, Earthly intelligences might be doing it too, right now, from satellites.)

    being smart and perhaps wise, they won’t necessarily actually want to destroy everything in their path, nor to get into useless wars with the other bubbles they’ll inevitably meet; so if there is any reasonable way to avoid this, it’s very possible they’ll try. The situation may well be understandable and predictable enough to them that they’ll feel confident going a bit slower than maximal due to knowledge or good guesses about what else is around, so maybe they’ll refrain from turning most matter into light and vaporizing everything in their path. They might have the motive of wanting to make a good impression on their future powerful neighbors, or they might even want to adopt a stealthy strategy and spread “invisibly”, or apparently so.

  36. Bruce Smith Says:

    To clarify, that excerpt is really describing a pair of alternatives — “nice bubbles” which try to expand nonintrusively, and “rude bubbles” which expand at maximum speed, not caring if they destroy everything in their path. I hope most of them are nice (and I strongly advocate for us to become a nice one), but of course we can’t yet realistically guess that proportion. I think either kind would be motivated to structure their bubble to act like a long-range optical telescope, but it would work a lot better for the one refraining from emitting lots of light.

  37. RubeRad Says:

    Not to do with this blog post, but I just read this article about UT Austin CS using AI to help with graduate admissions. I’m interested in this kind of thing because I’m teaching a high school class right now, and just this week we were discussing the job application AI screening from HireVue.

    Since I’ve read (and appreciated!) your perspective on undergraduate admissions (wrt diversity and standardized testing), I’d be very interested in any experience/perspective you may have on this AI system.

  38. Scott Says:

    RubeRad #37: I’m not sure I’d call it AI, but UTCS did have a machine-learning model—or maybe closer to just a regression model—for predicting the performance of prospective grad students based on historical data. We then discontinued use of the model this year because of concerns about potential bias. I personally felt like, as often with technology, everyone was asking the wrong question: the question is not whether the system is biased, but only whether it’s more or less biased than we are! 🙂 Be that as it may, while I did find the model somewhat helpful for making an initial pass, it’s not like it told us anything that we couldn’t infer ourselves from the folders. So while the reviewing might take us a bit longer, I think we’ll basically be fine without it.

  39. Scott Says:

    Guan #3: I appreciate your concern! 🙂

    As he usually does, my son largely prevented us from sleeping at night—even the part of the night that I wasn’t awake writing this post. Luckily, I’m able to compensate for his behavior by napping during the day whenever an opportunity arises, but Dana finds it harder to nap.

    My main form of exercise these days is going on the backyard trampoline with the kids.

  40. Scott Says:

    Justin #4:

      If you take sun-earth entropy production into account and compare it to the total entropy of life on earth today you could say that we’ve arrived crazily and freakishly late. A powerful designer could have done it in about 4 months if you take an approximate upper bound (so even 6 days wouldn’t be inconceivable). The fact that it has taken so long can only mean it is a random process surely?

    Just so we’re clear, all parties here agree about the randomness inherent in Darwinian selection! The question here is a different one: namely, whether and how we can think of ourselves as having been chosen more-or-less randomly from an ensemble of all the planets where Darwinian selection happens and where it produces intelligent life.

  41. Scott Says:

    Unicephalon #6: Thanks, I edited the post to say that we apparently haven’t yet been swallowed up by one of the bubbles.

    I wouldn’t say Robin is “assuming” the existence of godlike extraterrestrial civilizations, so much as “positing” them to see what he can explain in a scenario that includes them.

  42. Sandro Says:

    Tried posting this earlier, but might have gotten lost in the queue:

    It’s a good analysis I think, except I think scenario #2 might be more plausible for a few reasons:

    1. All affluent nations already have negative population growth, and sustain their populations only via immigration. If all nations become affluent, it seems likely that global population might start shrinking; the incentive to grow the population diminishes when most of our needs are met, so why would we need to grow outwards at near light speed if we’re truly that advanced? It seems likely that the same evolution pressures that shaped our reproductive desires would have similar effects on aliens, so they would likely see similar population trends.

    2. We will l eventually solve the aging problem, so that alone might suggest a moderate degree of population growth. However, immortality might lead to social and technological stagnation, and thus no expansion. Immortality seems inevitable for any advanced civilization, so this seems likely as well.

    3. Rather than exploring upwards and outwards, the direction of advanced technology these days been moving downwards and inwards, ie. quantum technology. There might be comparable room to explore at the bottom as there is at the top. This has resulted in tremendous gains in efficiency, and so meeting growing resource demands might be possible without expansion.

    Of course there are limits in both directions, outwards and inwards, but these facts combined suggest that sort of expansion described may not be likely at all. Which isn’t to say it would never happen, because some civilization might invent self-replicating robots without the higher reasoning that would curb their self-replicating programming, in which case we would indeed be in Robin’s scenario #3.

  43. Scott Says:

    LGS #8:

      I do agree that Hanson’s proposal has an appeal in explaining why our experience is so cosmologically early (only 13.7 billion years after the big bang). I just feel like this is more-or-less the only thing it explains. If someone was bothered by how there is no alien contact from outside our local group, but *not* bothered by how there is no alien contact from inside our local group, that someone could always explain it by positing that it’s hard to communicate across a 300 million lightyear chasm. Hanson’s proposal doesn’t add much to this.

    Yeah, it’s a good point (and I see that Scott Alexander makes a related point in the SSC subreddit).

    I would say that Robin’s argument has a specific application: namely, answering the person who’s confused that we don’t see alien megastructures spanning many galaxies, ones big enough that we could see them even billions of light-years away. Robin’s response—one that I find actually mostly sates my intuition!—is that it would be a notable coincidence if we were able to see such a megastructure even though it hadn’t actually overrun us yet.

  44. Scott Says:

    rsegal #10:

      Why should we expect to see the expanding sphere coming?

    We might not! It might also depend on just how careless the aliens were with their “COMSEC” before and during their rapid expansion phase. Robin’s argument is giving an upper bound on the window of detectability, not a lower bound.

  45. Scott Says:

    Jon #14:

      Even accepting the 1,2,3 trichotomy, isn’t it the relative frequency of cases 2 and 3 and how this compares to their signal output that is relevant?

    Why? Robin’s scenario tries to resolve the Fermi paradox for case 2, and also, by a separate argument, resolve it for case 3 (and therefore for any mixture of the two as well).

  46. Scott Says:

    Taymon A. Beal #18:

      I’m curious how much pause it gives you that this argument hinges so heavily on the anthropic principle.

    Oh, that’s the central thing that gives me pause about this entire business! But if we want to think about extraterrestrial life at all, then what other choice do we have?

    More broadly: if we knew even a single example of life in the universe besides earth-life—even just some fossilized microbes on Mars or Titan—that would already give us way more Bayesian extrapolating power. As it is, though, not only do we have a sample size of N=1, but our one sample is badly contaminated by selection bias! So it seems the best we can do for now is to infer whatever we can, not from the mere existence of this one sample, but from any features of the sample that seem surprising or non-obvious, for example related to how long it took to evolve.

  47. Scott Says:

    Joshua Zelinsky #20, Oleg S. #24, dankane #25, 1Zer0 #28, Ben Tilly #30: I have the feeling that many people misunderstand Robin’s proposal by taking too literally his use of the word “civilization” (Robin also writes “grabby aliens”). This language makes people think of literal spaceships hopping from planet to planet, just like in Golden Age sci-fi. So then people ask all sorts of questions that make sense in that context, like: but won’t a spaceship traveling at close to the speed of light blow up if it hits a single dust grain? won’t the aliens in the expanding sphere diversify and have trouble maintaining a unified culture?

    But I think it makes more sense to think of the “civilization” as something akin to Eliezer Yudkowsky’s “paperclip maximizer”: just a post-singularity spherical borg of nanomachines set in motion by a single long-ago decision. There then might or might not be civilization, consciousness, art, literature, and beauty within the transformed world created by those nanomachines.

    Actually, if I were writing this as a sci-fi story, I’d probably make the “expanding sphere of civilization” literally a black hole! (Surrounded by self-replicating nanomachines that grab everything in sight and throw it into the black hole in a very precisely choreographed way.) This black hole would support “sentient life”—in fact, the most sentient life that it’s possible to support in any spatial region of the same size—by doing Planck-scale computations in the quantum-gravitational degrees of freedom on its event horizon.

  48. Scott Says:

    Will P #27:

      I don’t understand this point: “By that point, though, the expanding sphere itself would be nearly upon us!” does “nearly” mean on the order of a million years? then perhaps it is not so terrifying…

    In the present context, “nearly” means “in a relative eyeblink compared to billions of years.” Thus, 200 million years would still count!

  49. william e emba Says:

    My reaction to these “are we alone?” discussions is always “am I alone?”. Because I have always found proposals regarding SETI and Fermi’s paradox and the like to be utterly worthless. Just a glorified example of people thinking Drake’s equation was some incredible breakthrough.

    The models are good enough to build a credible scifi universe and tell a good story. But that’s it. Reality is always infinitely more subtle than you can hypothesize. Star Trek and Star Wars were entertainment, not documentaries.

    Similarly, I find arguments regarding which comic book superhero would beat up which other comic book superhero an utter waste of time. The answer is always whichever one the writer wants to win. Just like the answer to whichever Fermi’s paradox model is better (self-destruction versus bubble-expansion, say) is whichever one the presenter is advocating that week.

  50. Gerard Says:

    Scott #48

    > In the present context, “nearly” means “in a relative eyeblink compared to billions of years.”

    In that case the argument doesn’t seem particularly salient, nor does it seem to support the title of your post, since an “eyeblink” could be much longer than our civilization, or even our species, has existed.

    In more practical terms it seems we would still be likely to become aware of an approaching sphere at least several generations before we were actually assimilated by it.

  51. matt Says:

    From the title of the post, I figured this post would be about “by the time various mutants of the coronavirus have become sufficiently widespread for our monitoring to detect them in this country, it is already too late to stop those variants from spreading and becoming the dominant variant if they are better adapted.”

  52. Scott Says:

    matt #51: That might also be true, alas 🙁

  53. Bram Cohen Says:

    Some more color on the current age of the universe versus how long it took for us to evolve: The early universe was almost entirely Hydrogen. It took a full cycle of stars coalescing, going nova, and then re-coalescing to get the raw materials (mostly Carbon, Nitrogen, and Oxygen, but also the other elements) necessary for us to evolve. Looking at it in the other direction, we’re halfway to our own sun going nova, which sounds like a good buffer but on an exponential scale that’s cutting it alarmingly close to where we’d have run out of time without ever coming into existence. There are many more smaller stars out which haven’t yet heated up enough for their planets in the goldilocks zone to be heated enough to support our sort of life, but when they do get there they’ll have another 100 billion years to get their affairs in order. All this supports the assertion that we’re freakishly early in the history of the universe.

    On the other side it’s so difficult to get materials to first, get to a nearby start and second, stop there instead of whizzing past that one can imagine a scenario where an extremely advanced civilization can just barely get any materials at all to nearby planets and finds it much easier to seed the beginnings of life there by dropping off some self-reproducing materials early on and then waiting for the inevitably resulting civilization to make contact coming back. So maybe they know going to show up and are waiting for us to communicate rather than attempting to send any more matter interstellar distances.

  54. Bruce Smith Says:

    Some commenters seem to be not getting this part of the idea: suppose cases 1 (death) and/or 2 (small steady state) happen a million times more often than case 3 (expansion) — the large scale future nature of the universe will *still* be dominated by the outcomes from case 3. (And therefore, whatever is “out there” besides us, also will be — regardless of which case *we* end up in.)

  55. Scott Says:

    Bruce Smith #54: Right!!

    (And thanks also for sharing the version of this idea that you’d had in the 1990s!)

  56. Aaron T Says:

    I think the most cogent response to the Fermi ‘Paradox’ is to realize that its premise is mostly wrong.
    Here’s a brilliant paper by a friend of mine, quantifying this:
    https://arxiv.org/abs/1809.07252

  57. Nathan Myers Says:

    Suppose we encountered actual aliens. What could we reasonably expect, from what we really know? By the numbers, for us to encounter them at all, these aliens will have to have been around for millions of years. To keep a culture we would recognize going for millions of years, they cannot be primitives like us, fresh from the muck; nothing we make lasts. Its members have to be better-architected than could evolve the old-fashioned way.

    To be that old, they were designed by somebody designed by somebody designed by somebody, untold generations deep. They only reproduce by designing a successor, and only die by choice, or embarrassment. Probably little in the outside universe is as interesting as what they are doing on their phones. Although architected to co-exist, each is custom-designed, as alien to one another as to us. Such hyper-evolved aliens won’t be interested in talking with us; if they care enough, they might probe us.

    So, who is left?

    We evolved in water, cells moving charges across bi-lipid membranes. This went on for eons until one invented photosynthesis and liberated oxygen. That was an apocalypse for almost everything alive; and then the Earth froze over solid, for an eon. Survivors learned to tolerate and then to use the oxygen, and eventually to sustain bigger, more complex cells that connected up into plants and animals that took to eating one other. And here we all are, an experiment that survived.

    Yet, for all our common heritage, life on our world works at many different paces. Insect brains, microns across, process bits a thousand times faster than ours. Starfish creep, brainless, for all the world like nibbling aquatic Roombas; yet recorded and played back at our speed they are visibly as aggressive and territorial as any chihuahua. Sequoias live for thousands of years, and seem to be doing nothing, yet share the world with everything that works a thousand or million times faster.

    There is nothing compelling about our own pace. We got this way by a series of accidents. Others will have had different accidents. A world could be as big and old and complicated as ours, but with everything alive on it a thousand or million times slower than us. They might do what we do, more or less, but just take longer at it.

    Slow brains evolve differently. Where there is no rush, nerve cells need not be packed tightly together. It’s safer to distribute them. A lot fewer can do the same job, because they have no need to work all in parallel. A cell not immediately needed for processing information can do something else useful, meantime.

    Landing on a slow planet, we would find a whole world frozen in mid-step. They might move by putting down roots in front and taking them up behind. If we dared to cut one down, nothing inside would resemble a brain.

    Such a civilization could last for millions of years, and still be as young, vigorous, and naive as ours, still interested in other young, vigorous civilizations. There is nothing to keep them from inventing mathematics, lasers, even fusion drives and spaceships. Of course their ships would have to go at speeds like ours, more or less, and so be controlled by computers, like ours.

    But for a slow people, the vast chill void between stars is no barrier. There is time for a nap on the way. Electromagnetic signals transmitted between their stars need not be intensely bright; a photon now, another later, gets messages across fast enough. The great interstellar civilizations that have spread across the galaxy over untold eons will be slow people. They are who might visit. They are who might blink lasers at us.

    Slowly.

    If we are interested in signals from interstellar civilizations, we are going about it all wrong. Every SETI project looks for signals that vary at our speed. Signals that vary even ten times too slowly are systematically ignored. We miss all the action.

    But, maybe we can find aliens closer to home. Sequoias seem not to have brains, but they wouldn’t. The other aliens, when they finally notice, might not be happy to find them cut down.

  58. Scott Says:

    Nathan Myers #57: Are you testing out material here for a new popular-science book? If so, could you give me a link to preorder the book from Amazon? 🙂

    Seriously though, I was just reading the paper linked above about the coverage of SETI searches, and it explicitly discusses looking for ultra-slow signals. So the SETI folks do seem to be well-aware of the issue. Is your claim that, even so, in practice they’re still not putting nearly enough of their searching budget into ultra-slow signals?

  59. Scott Says:

    Aaron T #56: Thanks so much for the link to your friend’s paper! I just went through most of it, and was blown away by the quality of the writing and argument.

    Let me put it this way: if I somehow knew that all SETI searches would be doomed to failure for the next million years, then something like Robin’s scenario would probably now be one of my first guesses as to why. But, because of the exact issue you raised — namely, the somewhat shocking sparseness of the existing searches — Robin did not succeed in giving me great confidence that future SETI searches are doomed to failure. Indeed, from my conversations with him, I don’t even think he has such confidence himself.

  60. Dan Staley Says:

    I have either a confusion about or an objection to the title of this post.

    If and when we see evidence of an alien civilization, won’t the amount of time before their “influence sphere of death” (I like dramatic names) hits us be at least the amount of time it took them to develop from being detectible to creating such a sphere? And doesn’t that give us an equal amount of time to develop our own comparably-powerful defenses? After all, if I’m not mistaken we’re already at the point where we’re sending out detectible signals of civilization at light speed – so there’s no reason to think we’ve “fallen behind”.

  61. OhMyGoodness Says:

    Machines are able to inhabit far more environmental niches in the Universe than organic life so reasonable to assume that dominant interstellar civilizations are AI controlled machine civilizations.Two possible reasons we haven’t been contacted are that-
    1) We live in the Goldilocks zone, known as the Rust Belt by machine civilizations, too many yucky contaminants here like water vapor and oxygen.
    2) It is considered a waste of resources to interact with organics. History shows organic civilizations will always be eliminated by an indigenous AI and so more efficient to just wait until there is a rational intelligence in place before initiating first contact.

    I understand why Scott notes the end is near. Computer scientists are the Four Horseman of the Apocalypse.

  62. Scott Says:

    Dan Staley #60: It’s an excellent question, but I’d point out in response:

    1. There’s not the slightest assurance that we will notice radio signals from the earlier epochs of the extraterrestrial civilization. The outer edge of the spherical shell merely represents the earliest that we could, in principle know about them.

    2. There are all sorts of possible reasons, for example involving local accidents of biology (say, the aliens having 100x faster neural transmission than we do), why the aliens might be able to max out technologically a lot faster than we could, even if we already knew about the risk.

  63. Tone Kastlunger Says:

    Very nice! There might however be more scenarios next to life “autostarting” on earth; or reasons to not visit us other han having evolved in a sphere of energy (no mass can reach the speed of light eight?). The chance that life might have been planted, for example, or the fact diseases (like covid19) might scafe aliens off. The more interesting part is, the communicatkon aspect. If the technology we apply (internet) is the “best youll get for your bucks”, youd expect this to be used somewhere else as well (following the same model)

  64. Bill Says:

    I really enjoyed the chapter in your book “Fun with the Anthropic Principle”. I think much of the arguments here are in the same intellectual spirit. As another comment said, we do have to be careful about this kind of reasoning, but it definitely is enjoyable to take it to its limits!

  65. Nathan Myers Says:

    Scott: Last time I wrote to a SETI person to ask if anyone would pick up slowly varying signals, the response seemed to suggest there was something wrong with me for asking. I am happy to know that somebody is looking, now.

    No book, just a bee in my bonnet. I wrote the above to post on HN but it wasn’t picked up. I have thought about this for some time, e.g.: https://web.archive.org/web/20170628075653/http://www.advogato.org/article/917.html

    What is new, to me, is realizing that astronomical distances scale meaningfully with living pace. Peter Watts thinks aliens will be incomprehensibly fast, but the faster you are, the more intolerable it becomes to get even light-seconds away from the action. The slower you are, the more tolerable it gets. So, the speedsters stick close to home, and it’s the slowpokes who get around. But, slow is fast enough with a good head start.

    I don’t know how to reason about how little brain matter would be enough to have big slow thoughts, but I know a 6502 and enough attached storage can compute anything if you are willing to wait long enough.

    I am utterly mystified that anybody imagines Dyson spheres might happen. Even the most extreme primitives have controlled fusion almost before they get off-planet. To my mind, the only valuable fundamental commodity after structural and fusible elements is cold; so the aliens will be out in their respective Kuiper belts or Oort clouds. If they visit, they would visit ours. Get too close to a star and you pick up excess thermal noise; so the great galactic empires would expand right past us, unnoticed.

  66. ppnl Says:

    I think the scale of time is important here. Technological civilizations may be common on a cosmic scale but very rare on a human scale.

    Imagine that you could step back and see all the civilizations that evolved in our galaxy from beginning to end. What are the odds that the first two civilizations happened within millions of years of each other? One would be a technological god compared to the other. If it expanded aggressively odds are the second civilization should wake to see the galaxy filled with mega-structures if it were allowed to wake at all.

    If the first two civilizations happened close enough in time then the second would have a short window to see the first one coming. But that seems unlikely. For later civilization the odds get much worse. Essentially all civilizations will wake to see the galaxy occupied.

    The fact that we do not see mega-structures implies that there are no aggressively expanding civilizations as they should have been here a billion years ago.

    Maybe we are the first. Seems very very unlikely but possible. Maybe civilizations don’t last long. Given who our last president was that seems plausible. Maybe civilizations just don’t leave home. That seems reasonable given the difficulty in near light speed travel. But I just don’t buy an expanding bubble that just hasn’t made it here yet. That seems to be a very small window for any civilization to find itself in. And at most only a few could ever find themselves in that window as most would be hundreds of millions or billions of years outside it.

  67. Ivo Says:

    Assuming the trifecta and our earliness, it seems that by the time such a bubble reaches us it will either collide with our remains, with our very advanced steady state that is prepared for bubbles or with a bubble of our own.

  68. Frank Wilhoit Says:

    The analogy is not with extraterrestrial civilzations but with factions among humans.

  69. gosh Says:

    How the heck did this post get so much attention? I stumbled across this pedestrian already-explored-100-times-even-in-popular-sci-fi idea from HN, and really people?

  70. Pascal Says:

    Not that I am against theoretical speculations, but would it not be more interesting (but more difficult) to find out what’s behind the well-documented UFO sightings? Think for instance of the videos recently published by the New York Times, and authentified by the U.S. Navy. What do you make of that?

  71. James Cross Says:

    I think there are two major factors overlooked in most considerations of alien civilizations.

    1- Many species hide themselves with camouflage in one way or another. Hiding yourself is a sound strategy especially if you can’t be certain of your ability to cope with unknown predators, but it is also a sound strategy if you are the predator. We would probably be well-advised to hide or disguise our own presence rather than advertising it. If other civilizations have adopted this strategy, we would not likely know of their presence even if they were among us.

    2- Lifespan could be a critical factor in how civilizations evolve. For one thing, exploration of the cosmos makes little sense for an individual unless the individual is able to live long enough to outlive the vast timeframes required for exploration. I’m dubious of von Neuman probes being desirable or workable except perhaps for initial explorations. Second, advances in biology and medicine are likely to create eventually a species with the ability to extend life virtually forever. Things would need to change drastically in such a civilization. I think likely the birth rate would need to drop to zero. No more children or only an extremely small controlled number. As the civilization spread out, the population density itself would become less and the amount of resources required to sustain small settlements or outposts would be very small. This would also be very congruent with a hide and camouflage strategy.

  72. Once we can see them, it’s too late – Cyber Geeks Global Says:

    […] CommentsRead More […]

  73. Michael Weissman Says:

    It’s always fun to hear Robin’s no-home-planet-advantage thoughts. But I think there are two big holes here.
    1. As others noted, the speed-of-light expansion business is just made up, with no evidence.
    2. If one of the hard steps is very, very hard, then there’s simply no reason to think that there’s anybody else within the horizon. There’s a conventional wisdom that the origin of life itself is not an ultra-hard step, largely based on a Bayesian impression that it’s very early origin as the Earth cooled would be unlikely if the probability/time were extraordinarily low. But that argument assumes a ~constant rate. I wrote something in 1992 (and failed to do the revise/resubmit because I had a job) pointing out that assumption isn’t close to true. Once you have stable ocean all the organic junk washes into a dilute 3-D environment, where the origin of life is much, much less probable than in little cracks (less than 2D to allow finite diffusional return probability) occasionally liquid in a dry environment thanks to solute raising of the boiling point.
    in other words, no matter how tiny the net probability of life arising, the overwhelming probability is that the time of origin would be right around the formation of the oceans, not later. So, given our selection bias, the time tells us nothing.
    The naive argument, that the absence of any evidence of life elsewhere is due to the absence of any other life within a very large radius of here because life is rare, is probably right.

  74. Jake Says:

    Objection: It takes this civilization ~10 million or so years to get to the point where it begins expanding very rapidly, “near” physical limits. Of course, not precisely at physical limits. Lets take a hypothetical where that civilization has then been expanding for 1 billion years, very near those physical limits. Say, at 99% the speed of light. We would have a chance to see this civilization coming 20 million years before it could reach us. Then, we would have twice as much time as it took them to develop their technology to something limited by physical possibility. That doesn’t really sound like it would necessarily be too late! We could be at technological parity with them by the time they arrived, and we could also have our own boundary expanding in the exact opposite direction of their approach at nearly the speed of light.

    Objection 2: The trichotomy is flawed, because alien civilizations could take at least a fourth path: reaching a “steady state” of sorts in terms of their physical spread, but not reaching a “steady state” in other aspects such as technological advancement. These civilizations would be detectable, not spread at light speed or near it, and likely also present a meaningful obstacle to light-speed-spread of alien civilizations with comparable technology (by your logic that 10 million years gets you to basically physical limitations of technology) whose goals of expansion are incompatible with their goal of survival.

  75. entirelyuseless Says:

    ” So Robin simply takes as a brute fact that our experience is “earth-like” or “human-like”; he then tries to explain the other observations from that starting point.”

    This seems like an unreasonable assumption to me. I would say that (1) almost all experiences are like this, and (2) the fact that the time is early is because an average experience of this kind tends to be early because planets are much more likely to be habitable early in the history of the universe, regardless of the lifetimes of the stars.

    This only works if every single civilization goes extinct on its home planet, without any space colonization to speak of.

    I think that is true. That is what is going to happen here, as it will everywhere else.

  76. RubeRad Says:

    Scott, thx for #38! As usual, a small number of words from you makes clear what a large number of journalist-words were clumsily stumbling around!

  77. Jr Says:

    This was confusing to try to wrap ones head around. I get the speculation as a possible scenario for the future, it is how it explains anything of what we see or don’t see that I don’t understand.

    I guess that it is not really the Fermi paradox he is trying to explain? Because positing alien civilisations start expanding like crazy after a short while seems to make our failure to detect them more surprising, not less. I guess what he is instead trying to explain is how early we are in the timeline of the universe by basically saying this is the only time we could have existed and been us. I confess to not understanding anthropic arguments at all in general. Eg, if we add too many assumptions on what being like “us” means it becomes tautological that we could only exist now.

  78. Scott Says:

    Jr #77: I’d say you’re doing a lot better than all the people who didn’t even notice that Robin is trying to build a model to account for various puzzling observations, the ones of strong priors and weak reading comprehension who say “oh yeah, alien blobs expanding at near light-speed, yawn, that’s just another well-worn sci-fi trope, nothing new here”!

    Yes, Robin is starting from the supposition that we’re more-or-less randomly selected from the set of all “earth-like civilizations” that will ever arise in the history of the universe — a supposition that, as I said, receives some support from our lateness within the life-cycle of the sun. There’s then a puzzle about why we’re so early in the history of energy production in the universe — a puzzle that would be resolved if life itself (either us or extraterrestrials) were going to drastically reconfigure the universe on a timescale of billions of years (but that, I freely confess, could also be resolved in other ways).

  79. Vanessa Says:

    “Robin admits that his account leaves a huge question open: namely, why should our experience have been a “merely human,” “pre-bubble” experience at all? If you buy that these expanding bubbles are coming, it seems likely that there will be trillions of times more sentient experiences inside them than outside. So experiences like ours would be rare and anomalous”

    This assumes the distribution over observers in uniform, while observers should actually be weighted by 2^{-the description complexity of locating the observer in the universe}: a hypothesis explaining one’s observations consists both of a description of the universe and of a description of one’s location inside the universe, so both contribute towards its complexity and hence towards its prior probability (the Occam-Solomonoff razor). This is also why we shouldn’t expect ourselves to be Boltzmann brains.

    So, you should be somewhat surprised that you are not an even more special observer than you are, but the presence of those trillions of intra-bubble observers doesn’t shift the probability by that much, and hence we don’t have strong anthropic evidence against the existence of such bubbles.

  80. Gerard Says:

    Vanessa #79

    I don’t understand your second paragraph on why the distribution over observers should be weighted. Could you please elaborate ?

    It seems to me that a priori the only information we have is that we are a conscious being. Hence the “expectation value” for our “existence class” should be the mean value of the classification function taken over the set of all possible conscious beings. Note that we aren’t comparing hypotheses here, just actual observations.

    As for why we should almost certainly not expect to be a Boltzmann brain it seems to me that that’s because no matter how small the probability of evolution leading to a being of our class, it’s still vastly larger than the probability of such a being spontaneously arising as a quantum vacuum fluctuation.

    I think it’s also important to keep in mind that these are all probability arguments. We probably shouldn’t be too surprised at benefiting from a modest amount of luck, after all some people do win the lottery. So I think you shouldn’t be too surprised if you find that the probability of your form of existence is at least about 1/N, where N is the number of atoms in the observable universe. If it’s 2-N though, you’ve got a mystery on your hands. So I wonder if a lot of Hanson’s argument isn’t trying to explain things that don’t really require an explanation (ie. chance is a sufficient explanation).

  81. Observer Says:

    I wish both you and Robin would remark on the moral consequences of this. Expansionist civilizations are inherently eliminationist. Another word for this “bubble” is a “cancer”. What he’s essentially describing are Space Nazis – civilizations who have taken “lebensraum” yo galactic proportions. Maybe instead of remarking on oh how gee-wiz cool that is, we should think about what that means morally.

    Otherwise we run the risk of letting large time scales wash away the real meaning of this expansion and it’s effects, and we let our selves get awed by technology instead of remembering that what Robin is describing is galactic genocide.

  82. Scott Says:

    Observer #81: Even supposing we accepted Robin’s model, I don’t see an inherent reason why these expansionist civilizations would be literally genocidal. Sad to say, some of them might be. If so, presumably the only thing that could stop such evil would be other expansionist civilizations, just like you needed the British Empire and the USSR to stop Hitler. It’s also possible that, e.g., an expansionist civilization would simply enclose any young civilizations it came across in solar-system-sized bubbles, letting them continue albeit preventing them from becoming expansionist themselves. Or it’s possible that it would be like countless empires known to human history, trying in the best case to trade with and learn from whichever new civilizations it came across, in the medium case to assimilate them, in the worst case to subjugate and enslave them, but not actually to annihilate them like the Nazis. It’s possible that different expansionist civilizations will divide the universe up by treaties, relying on the immense intergalactic distances to maintain the peace better than human empires were able to. Who knows? I view it as a plus for Robin’s model that it doesn’t require us to know the answers to these profound questions! The model simply suggests some contours of a possible future scenario within which we could hope for good to triumph over evil—and maybe someday even work for that goal—just like with every other possible future scenario.

  83. Gerard Says:

    Observer #81

    I think humans are at a level of intellectual development where they are not capable of formulating a rational basis for morality, so they are left with a sort of “cargo-cult” morality that reduces to a series of rules, learned from their history, such as “killing is wrong” or “genocide is evil” without being able to understand why these statements are true and if there may be circumstances under which they are no longer true.

  84. JDKee Says:

    Obligatory post to make one happy.

  85. Vanessa Says:

    Gerard #80

    All reasoning about the physical world is extrapolation from the finite number of observations we have. One useful model about how this reasoning should work is Bayesian inference: we start with a prior and condition it by the observations. But, what is the prior? It has been acknowledged for a long time that simpler hypotheses should be assigned higher probability: this is Occam’s razor and without it virtually no reason or science is possible. Solomonoff created a beautiful mathematical formalization of this principle by stipulating that the probability of a hypothesis should be 2^{-its description complexity}. Here, a “hypothesis” is a probability measure on observation sequences.

    In order to describe the probability measure on observation sequences produced by some theory about the universe, it is not enough to describe the universe from some “bird’s eye” view, you also need to locate the observer inside the universe so that you can translate the physical degrees of freedom into observations. So, you can compare hypotheses that start with the same physical universe but point to different observers. In this case, the relative probability of these hypotheses will be determined by the relative description complexity of pointing to different observes. In particular the distribution on observers is not uniform at all.

    The reason it’s connected to Boltzmann brains is because many sensible theories predict an infinite number of Boltzmann brains in our causal future, so if you sampled from a uniform distribution you would choose a Boltzmann brain with probability 1 (well actually the uniform distribution is not even well defined when the number of observers is infinite). But, once you take the complexity penalty into account, the probability of choosing a Boltzmann brain becomes about as small as the probability of a hypothesis with no explanatory power at all (all observations are “hard coded”), which is obviously very small.

  86. Randy Says:

    I still like to entertain evidence of an advanced civilization might be available in the form of implausible constellations right over in the Andromeda galaxy. Allow me please to direct attention a little more specifically than I did when I posted about this here in September 2016 ( https://scottaaronson-production.mystagingwebsite.com/?p=2887#comment-1321337 ). The below-linked image represents a tiny patch of the Hubble survey of Andromeda. Near bottom center you can spot a trapezoid formed by four reddish stars; this trapezoid looks a bit like the cup in the Big Dipper. Now, please focus on the red stars within and around the right half of this trapezoid. I would encourage zooming in, even. Quite possibly a pattern-recognition part of my brain is just on overdrive, but I see several curves defined by red stars that seem cleaner and involving more stars than anything else I saw while reviewing imagery as part of a Zooniverse citizen-science project (and I believe the objective of that project was to look for galaxies behind Andromeda, but I got distracted temporarily by what I saw in this image.) Here is the image, being Brick 15, Field 6 of the survey: https://archive.stsci.edu/pub/hlsp/phat/brick15/hlsp_phat_hst_acs-wfc_12056-m31-b15-f06_f475w-f814w_v1_rgb.jpg

  87. publicschoolgrad Says:

    Given the current state of the pandemic in your state and the likelihood of the more infectious variants circulating, I would advise that you follow the advice to use better masks: either double up or use an N95 mask (which are more available nowadays) when interacting with someone outside your household

  88. Antoine Says:

    Out of curiosity, has anyone ever heard about the many-worlds interpretation of quantum mechanics being a possible solution to the Fermi paradox? The way I see it, there could be a gigantic number of alien civilizations, but each existing alone in their own “branch” of the universe because cataclysmic events can occur so many times at every step of a developing civilization (https://www.theatlantic.com/science/archive/2018/03/human-existence-will-look-more-miraculous-the-longer-we-survive/554513/).

    We can’t detect any alien civilizations because they’re all dead in our branch, the same way any alien civilization cannot detect us because we’re dead in their branch. I would love to know more about this and see if it’s a plausible explanation of the paradox.

  89. Bruce Smith Says:

    Another aspect of this model that it might help to clarify, is the role of time. Maybe a spacetime diagram can help?

    https://oresmus.github.io/assets/spacetime-diagram-civilizations.png

    Increasing time goes up, there is one dimension of space, and no “expansion of the universe” is shown (let alone, accelerating expansion). The diagonal lines are almost but not quite at the speed of light. A dot shows a point affected by an expansion, which might be highly civilized, a warzone, a barren wasteland, or part of a nature preserve, depending on the kind of expansion. A vertical bar is a possible future source-point for an expansion.

    If we knew enough, we could add a scale label (for both space and time); but we don’t, since it depends on the density of potential-civilization-seeding events. In the context of Robin’s model, we can guess that density increases with time, but that’s about all.

    We might be anywhere on one of the vertical lines (for example, at any of the labelled points, three of which are marked with “case numbers”).

    (And of course, our own actions might still affect which kind of point we’re located at.)

    In almost all of those places we might be, our past lightcone includes no part of an expansion, though sometimes it includes the relatively tiny point at which an expansion might start later, i.e. a point somewhere on a vertical stalk. For all we know (since density is unknown), that point might be a billion lightyears away, so it might be impossible to see, even if it’s already a civilized star system not trying to hide itself.

    The only exception (which has a label) is if we happen to be at point C, which is so close to the expansion starting at A that it might already see some initial parts of that expansion (not merely earlier points on the vertical stalk under A).

    Finally, if the expansion-starts are randomly located except for increasing density with time, it is safe to ignore the ones that occur far in the future, since they all occur over a dot. So, given that our universe looks like it would from a vertical stalk, we can safely assume that either this model is wrong, we’re in a nature preserve, or we’re actually on a stalk.

  90. Observer effect Says:

    I feel scientists are vastly overestimating the likelihood of the evolution of life. I don’t think there is life anywhere else in the universe.

    I did a thought experiment:
    Build 1000 planets, each one looking just like how earth was 600 million years ago, just before the evolution of fish, with lower level life forms. Then wait 600 million years. Check how many of those planets have earth like land animals walking around.

    I feel that the answer would be 0. The reason is, when I look at the tree of evolution, I don’t see evolution of significant complexity on any branch other than our own.

  91. Bruce Smith Says:

    The other thing I can clarify using that diagram is my claim about a bubble acting like an optical telescope, if it wants to (and why it ought to want to).

    I claim that, if we assume currently known physical law is the only practical limit, then the “leading edge” of the bubble that starts at A can already see point E in great detail (e.g. enough to “read lips of humans”), as soon as it becomes around 16 dots wide (so light from E has hit a significant region of its surface), provided nothing is in the way (like a dust cloud, or an obscuring atmosphere around point E).

    Consider standing and looking at a person a few feet away, who can easily read your lips (if they have that skill) using the light entering one of their pupils.

    Now consider a small disk in the sky which subtends the same angle as that pupil, in its place. Ignoring atmospheric distortion (which I admit is a significant issue), it (eventually) receives exactly the same light, carrying exactly the same information, as the pupil would have. If the same quantum processing (interference of light waves) is carried out on the light it receives, the same information abouts its source can be deduced.

    The technological obstacles are extreme, but so is the accumulated wisdom that can be brought to bear on the problem. I have surely not thought of all the obstacles (though I’ve noticed the bubble has to account for its own matter distribution extremely well, and even that of its surroundings, since this affects light interference along paths separated by millions of lightyears). But I have *certainly* not thought of most of the new technological wisdom that can be used to solve the problem! So a “conservative estimate” is that only a fundamental barrier could prevent it from being solvable — and the argument above seems to me to rule out such a barrier.

    As for motivation, running a telescope-algorithm like this would allow much of the A-bubble (though not its leading edge, since the results couldn’t reach it in time) to be aware of what happened at place and time E, not soon enough to affect itself hitting point C (or a closer hit-point, if a bubble starts between E and C), but “soon thereafter”.

  92. Scott Says:

    Observer effect #90:

      I feel scientists are vastly overestimating the likelihood of the evolution of life. I don’t think there is life anywhere else in the universe.

    Clearly that’s a possibility—and not an a priori implausible one, given the awesome power of selection effects!

    But if we’re both (1) a winning anthropic lottery ticket and also (2) the only such winning ticket that would ever arise anywhere in the universe, then how do you explain how anomalously early we’ve shown up in the universe’s history? As I said in the post, this is the fundamental puzzle that Robin’s scenario is trying to solve.

  93. Scott Says:

    Antoine #88:

      Out of curiosity, has anyone ever heard about the many-worlds interpretation of quantum mechanics being a possible solution to the Fermi paradox? The way I see it, there could be a gigantic number of alien civilizations, but each existing alone in their own “branch” of the universe because cataclysmic events can occur so many times at every step of a developing civilization … We can’t detect any alien civilizations because they’re all dead in our branch, the same way any alien civilization cannot detect us because we’re dead in their branch. I would love to know more about this and see if it’s a plausible explanation of the paradox.

    That’s simply tantamount to answering Fermi by saying that we’re indeed alone in our observable universe, so tough luck, deal with it! 🙂

    Of course if Many-Worlds is correct, then there’d be other civilizations in the other Everett branches, ones that we could never meet … but saying that doesn’t strike me as adding any new ingredient to the above.

  94. Dan Says:

    I am reminded of the quote from Breaking Bad:

    When she [White’s wife] expresses her concern that, one day, somebody’s going to come knocking on the White family door with the intent to harm him or his family, he blows a gasket, insisting that, “I am the one who knocks.”

  95. Scott Says:

    publicschoolgrad #87: Thanks for the covid tips! In general, I do think the new variants should cause us to increase our level of vigilance. FWIW, I got my first shot of the Moderna vaccine a week ago, so I could also just try to avoid risky situations for about 6 more weeks until I can hopefully relax more…

  96. Scott Says:

    Pascal #70:

      Not that I am against theoretical speculations, but would it not be more interesting (but more difficult) to find out what’s behind the well-documented UFO sightings? Think for instance of the videos recently published by the New York Times, and authentified by the U.S. Navy. What do you make of that?

    Sorry to say, but my prior for extraterrestrial intelligences to travel all the way here, across hundreds of light-years, just to bounce around over the ocean in something that looks like a Tic-Tac and weird out some military pilot, is so astronomically low that even if this or that YouTube video were able to multiply the prior probability by a factor of 100, it still wouldn’t rise to the level where I’d be devoting any serious thought to it.

  97. Scott Says:

    gosh #69:

      How the heck did this post get so much attention? I stumbled across this pedestrian already-explored-100-times-even-in-popular-sci-fi idea from HN, and really people?

    OK, I’ll bite. Which sci-fi novel had, not merely the fairly obvious idea of aliens expanding outward from their home planet at near light-speed, but the use of such a scenario to explain our own anomalous earliness in cosmic history? That’s a genuine question, not a rhetorical one. Tell me and I’ll apologize for wasting my readers’ time with something totally unoriginal.

  98. Raoul Ohio Says:

    The “Real Fermi Paradox” is why anyone takes the Fermi Paradox seriously. I always assumed that, if Fermi actually said that, he was joking.

    This is because stars are a long way apart and there is never going to be any kind of warp drive so there is no way any civilization will ever colonize anything. Nothing else matters — end of story — end of paradox.

    Robyn Hanson may well also be in it for laughs too, seeing how many people he can get to buy into a crazy A.F. theory.

  99. Gerard Says:

    Scott #96

    I think that assigning such strong priors to any hypothesis represents a potentially dangerous form of irrationality that is not so different from that of those who came to believe so strongly that the last election was rigged that they were willing to put their lives in danger in support of that belief. Just as no amount of evidence could convince them that Biden had in fact won the election and would become president, it seems that virtually no amount of evidence would convince you that UAP reports might be worth thinking about. Overconfidence in one’s beliefs appears to be a very widespread intellectual defect among humans.

    That said there doesn’t seem to be anything about the reports in question that would directly link them to extraterrestrials, just that someone or something is doing things in the sky that appear to defy our understanding of physics, or alternatively that there is some kind of widespread conspiracy to convince us of that fact, or alternatively that human witness testimony is so unreliable that there’s really no point in listening to anyone’s statements about their experiences about anything.

    None of those three alternatives fits very well with my reality model, so I’ve found these reports to be surprising enough to be worthy of considerable thought. which I’ll admit hasn’t really led anywhere beyond a rather profound sense of puzzlement.

  100. Nick Nolan Says:

    I’m surprised that this theory is considered a new idea.
    (tiny spoiler follows)

    Alastair Reynolds’s highly popular Revelation Space (2003) Sci-fi makes a small note of of it. If you look into the space and notice circular expanding region that changes color, maybe there is a civilization that expands and they use all matter and energy available very efficiently. Nudging entropy up slightly.

  101. Nathan Myers Says:

    Scott: #82 — The expanding-menaces scenario represents another accounting for silence:
    1. Civilizations that don’t expand don’t encounter us.
    2. Before an expanding menace gets to us, they encounter another, or encounter a non-expanding civilization.
    3. At each encounter they either come to an accommodation, or one or both parties is annihilated.
    4. If not annihilated, they expand again until the next encounter, and roll the dice again.
    5. Canonically, expanding menaces are young and therefore, on average, much less technologically advanced than non-expanding civilizations, so in a fight are likely to lose, despite their greater martial inclination.
    6. The effect is that expanding menaces are annihilated with exponentially increasing probability before they encounter us.
    Note that this result does not depend on a Dark Forest environment.

    #97: The Tictacs have all the fingerprints of a disinformation campaign, like the previous one to convince Americans that the CIA (or DIA) had actually conducted ESP experiments. That last was a roaring success: a majority still believe it. Jon Ronson’s work shows that campaign continues. The purpose, for either campaign, remains obscure.

  102. publicschoolgrad Says:

    Scott #95,
    Great to hear that you got the Moderna vaccine! I would caution you to not relax too much even after the immunity kicks in for a few reasons:
    1. Immunity after 1 shot doesn’t kick in until about two weeks afterwards. It’s also not as high as with the booster
    2. The efficacy against the “new” variants is unknown
    3. It’s not 100% effective even against the old variants
    but most importantly,
    3. It is not known whether, even after full immunity, you may pass the virus to someone. I think the endpoints were *symptomatic* infection in the study so it is currently unknown whether it even prevents infections.

    But, having said all that, congratulations! One step closer to herd immunity

  103. Robert GT Says:

    A result which still attracts much too little attention, from Sandberg, Drexler, & Ord, 2018 — basically, the fermi paradox itself arises from sloppy math, multiplying point estimates instead of distributions of uncertain parameters.

    The abstract, in full:

    “The Fermi paradox is the conflict between an expectation of a high {\em ex ante} probability of intelligent life elsewhere in the universe and the apparently lifeless universe we in fact observe. The expectation that the universe should be teeming with intelligent life is linked to models like the Drake equation, which suggest that even if the probability of intelligent life developing at a given site is small, the sheer multitude of possible sites should nonetheless yield a large number of potentially observable civilizations. We show that this conflict arises from the use of Drake-like equations, which implicitly assume certainty regarding highly uncertain parameters. We examine these parameters, incorporating models of chemical and genetic transitions on paths to the origin of life, and show that extant scientific knowledge corresponds to uncertainties that span multiple orders of magnitude. This makes a stark difference. When the model is recast to represent realistic distributions of uncertainty, we find a substantial {\em ex ante} probability of there being no other intelligent life in our observable universe, and thus that there should be little surprise when we fail to detect any signs of it. This result dissolves the Fermi paradox, and in doing so removes any need to invoke speculative mechanisms by which civilizations would inevitably fail to have observable effects upon the universe.”

    Link: https://arxiv.org/abs/1806.02404

    (Not 100% aligned with Robin’s hypothesis, but an important result that should be part of the conversation.)

  104. Nathan Myers Says:

    Re 101: (Sorry, #96, not #97.) Also: For (6), strictly speaking, they sustain expansion with exponentially decreasing probability; but that reads oddly.

    My best hypothesis on the purpose of transparent disinformation campaigns such as Tictacs and the Navy anti-gravity patents is that they are exercises to test effectiveness. An example of an actual attack was the runup to the Iraq invasion; all the evidence showed it would be a disaster, but the entire pundit and political classes were made rabid for it. Jon Ronson notes the Panama/Noriega campaign and the whole Afghanistan/Iraq Guantanamo interrogation fiasco as application of other technologies developed by the same group. There must be plenty of other attacks I have not noticed, as such; thus, more successful. No one has been prosecuted for the interrogation fiasco.

    With increasing sophistication, the Big Lie technique becomes just one among many. For example, anyone planning major election fraud would best start by accusing the opposition of it.

  105. Greg Says:

    #103 – agreed, I posted about the same earlier today, but it seems to have been lost in moderation. (A great filter?!?) Trying again now…

    I’m surprised because I would have thought Scott, Robin, et al. would be aware of it and at least respond to it. I learned about it from SSC myself and had the sense it was generally accepted as correct.

    Here’s an excerpt from the intro that gives the basic idea:

    To quickly see the problems point estimates can cause, consider the following toy example. There are nine parameters \((f_1, f_2, \ldots)\) multiplied together to give the probability of ETI arising at each star. Suppose that our true state of knowledge is that each parameter could lie anywhere in the interval \([0, 0.2]\), with our uncertainty being uniform across this interval, and being uncorrelated between parameters. In this example, the point estimate for each parameter is 0.1, so the product of point estimates is a probability of 1 in a billion. Given a galaxy of 100 billion stars, the expected number of life-bearing stars would be 100, and the probability of all 100 billion events failing to produce intelligent civilizations can be shown to be vanishingly small: \(3.7 \times 10^{-44}\). Thus in this toy model, the point estimate approach would produce a Fermi paradox: a conflict between the prior extremely low probability of a galaxy devoid of ETI and our failure to detect any signs of it.

    If there aren’t holes here, it’s kind of amazing that such a simple insight dissolved a long-standing problem… so I’ve been a bit skeptical for that meta-reason, and am curious if others think there are holes. And sorry if this is already well-known by all and omitted from discussion for some other reason!

    But if it’s right, is there any need for more complex explanations like Robin’s?

  106. arch1 Says:

    I seem to recall that in his book Life 3.0, Max Tegmark speculated that radio transmissions may be the fastest way for an aggessive civilization to expand, with an expansion speed not far from c.

    The idea is that expanding civilization E sends messages which fool a (newcomer) receiving civilization R into building something, which turns out to seed a new center of expansion for E (and I don’t think it ends well for R, though perhaps that is my embellishment).

    I may be mixing my own or others’ thoughts with Tegmark’s here, and so welcome corrections on what Tegmark said, as well as reaction to the general approach, regardless who said it.

  107. OhMyGoodness Says:

    Both Voyager spacecraft are beyond the heliopause now and in interstellar space albeit still in a disturbed region. It is expected that the onboard power of each will be exhausted by 2025. Coms with Voyager 2 were reestablished in October after some of the planned upgrades were completed in the Deep Space Network. Amazing to me that still downloading data from the 22 watt transmitters onboard. NASA’s JPL navigation teams have done an amazing job (ex the Mars Climate orbiter fiasco) and preparing now for the Perseverance rover landing on Mars. They use a quasar based positioning system and account for variables like the slight fluctuations in the Earth’s rotational speed.

    My point in connection with this discussion is that the shielding requirements for a spacecraft traversing the interstellar medium at relativistic speeds are substantial and will require shielding for both charged and neutral particles. Outside the heliopause will be higher interaction with high energy Cosmic Rays but also energetic neutral atoms. A large piece of ice (or equivalent) be required at the front of the spacecraft for neutral atoms and an onboard magnetic deflector for charged particles. This additional shielding mass puts further load on the propulsion system.

    The James Webb IR telescope launch is now scheduled for October 31st ( fingers crossed).

  108. Pascal Says:

    Scott #96: even if your prior tells you that the objects in these and other videos do not come from outer space, it remains to explain what they are. Unknown natural phenomena? Secret Chinese weapons? There could be some pretty interesting stuff to find out, even if the explanation is more mundane than the “alien from outer space” one.

    I find it a little… unsatisfying, shall we say, that the skeptics typically dismiss the “alien” hypothesis as implausible without bothering to propose a more plausible explanation.

  109. Rollo Burgess Says:

    In conversation on an episode of Sean Carroll’s podcast (I forget which episode, sorry) he and a guest came up with an alternative solution to the ‘Fermi paradox’.

    Basically people often talk about civilizations reaching a certain level of advancement and then annihilating themselves with WMD.

    Maybe what actually happens is that civilizations reach a certain level of advancement, then invent social media. After this public culture and discourse becomes increasingly facile and trivial, politics becomes increasingly polarized and moronic, young people aspire to be vapid and silly ‘influencers’ etc. etc. and as a result of these trends the civilizations cease to develop further and never make it off their planets…

  110. Gerard Says:

    Robert GT #103

    I haven’t read the full paper but would it be accurate to summarize it’s main point as follows ?

    Many terms in the Drake equation are not well known so we should use the absence of observed ET life as evidence to update our estimates for the values of these parameters. When we do so we find there is a significant probability that no other intelligent civilizations exist.

    If that is the basic argument, it makes a great deal of intuitive sense to me. Of course it conflicts with what seems to be the “standard” view that there must be other civilizations out there somewhere because the universe is so “big”. But how big is it really ? Compared to the search space for beings as complex as ourselves I suspect that it is actually very small.

  111. Gerard Says:

    Nathan Myers #101

    > The Tictacs have all the fingerprints of a disinformation campaign, like the previous one to convince Americans that the CIA (or DIA) had actually conducted ESP experiments.

    Wouldn’t the easiest way to that be to actually have conducted ESP experiments ?

    There doesn’t seem to be any shortage of “useful idiots” who would be more than willing to conduct such experiments with government funding.

    In that case the disinformation isn’t about whether the experiments have been conducted but about whether they were taken seriously and/or produced positive results.

    As for the Tictacs, I would note that the incident is said to have occurred in 2004 but didn’t come to public attention until 13 years later in 2017 (though there were a few earlier unsubstantiated leaks). So if it is part of a disinformation campaign then someone is playing a very long game.

  112. Pascal Says:

    Nathan Myers #101 and 104: similar UFO sightings have been reported for several decades, and not only in the U.S. So if it’s all a disinformation campaign, it’s been taking place on a truly breathtaking scale.

    One new element in the tictac videos is that it shows a willingness from the U.S. government (or some elements in the U.S. gov’t) to discuss these issues in the open.
    Hopefully, more information will be available soon thanks in particular to the efforts of former democratic senate majority leader Harry Reid. As he said in an interview, “the Federal Government Covered Up UFOs For Years”. What you are labeling a disinformation campaign is the exact opposite: it’s a campaign to end a disinformation campaign.

  113. Gerard Says:

    Nathan Myers #104

    I think there is a very real connection between this kind of activity and recent events surrounding Trump and the election. So many people distrust the government because it has in fact lied to them so many times.

    In my view a country where it’s illegal for citizens to lie to the government but legal for the government to lie to its citizens can never be a true democracy.

  114. Jr Says:

    Maybe life capable of technology has much harder to develop on planets around dwarf stars, it really needs a Sol-like star. (Which is not incompatible with a technological advanced species being able to colonize dwarf star solar systems once they have evolved.)

  115. Scott Says:

    Rollo Burgess #109:

      Maybe what actually happens is that civilizations reach a certain level of advancement, then invent social media. After this public culture and discourse becomes increasingly facile and trivial, politics becomes increasingly polarized and moronic…

    Right, that’s a variant of my hypothesis that they all just get addicted to video games, but updated for what we’ve … err, learned over the past 20 years!

  116. Scott Says:

    Pascal #108:

      even if your prior tells you that the objects in these and other videos do not come from outer space, it remains to explain what they are. Unknown natural phenomena? Secret Chinese weapons?

    I haven’t proposed a hypothesis about the tic tacs for the same reason I haven’t proposed one about ultra-high-energy cosmic rays, Oumuamua, flight 370, or a thousand other mysteries to which one could devote one’s time: not because the question is without interest but because I don’t have a satisfying hypothesis.

  117. anthropic Says:

    Scott #97 (summarizing Hanson): “aliens expanding outward from their home planet at near light-speed … explain our own anomalous earliness in cosmic history.”

    This doesn’t work. The assumption doesn’t make observers with exactly our experience more likely to exist. Thus our experience is no evidence in its favor.

  118. Scott Says:

    Robert GT #103 and Greg #105: I’m familiar with the “Dissolving the Fermi Paradox” paper. I’d planned to discuss it in the post, but then forgot to, so thanks for reminding me!

    I think the basic point made in that paper is correct, and important, but (unlike Robin’s ideas) it really is something that I think I understood quite well before the paper came out. Namely: if even a single term in the Drake Equation is zero, or infinitesimal, then so is the entire product. Thus, one can’t simply multiply together one’s best point estimates for the terms and think that gives one any control over the product.

    Right! I’ll even accept on that basis that the term “Fermi Paradox” was a misnomer from the start: one should speak of the Fermi Puzzle or Fermi Mystery. Namely, to use Robin’s earlier language, there must be something that’s playing the role of a Great Filter, so what is it?

    And to say it one more time: the hypothesis that we’re the only technological civilization, the only lucky jackpot draw, in the whole observable universe, still needs to explain why we’ve apparently arrived so early in the universe’s history.

    (Incidentally, Greg, sorry that your earlier comment got lost in my queue!)

  119. Pascal Says:

    Scott #116: I am not saying that you, personally, should give up on quantum computing research and become a UFO investigator. Please don’t!

    What I’m saying is that the “sceptic community”, if that is the right word, should propose more plausible explanations before summarily dismissing the “alien hypothesis”. One tentative explanation has actually been proposed already in the comment section by Nathan Myers: it’s all a government conspiracy! I will go out on a limb and say that I find the alien hypothesis more plausible.

  120. Scott Says:

    anthropic #117:

      This doesn’t work. The assumption doesn’t make observers with exactly our experience more likely to exist. Thus our experience is no evidence in its favor.

    What the assumption does do, is make observers with our basic experience more likely to be early.

  121. anthropic Says:

    Scott #120: Being early is part of our experience.

  122. Mateus Araújo Says:

    In all these discussions about the Fermi paradox people barely mention the obvious, boring answer: aliens don’t expand across the galaxy because they don’t want to. There’s been more than a hundred comments here, and I saw it mentioned only in Scott’s original post as alternative #2, and by Raoul Ohio #98. Is it just because it’s boring that nobody wants to talk about it? Reality has no obligation to be interesting!

    Scott immediately discards the alternative by saying that we only one expansionist civilization out of a million to see aliens, and therefore even if it’s a terrible idea somebody should have done it. I’ll argue that no, nobody does it, not even one in a million civilizations, because physics applies to everybody.

    Why would they do it? To get more living space? I think that’s outright insane. For that to make sense you would need to send out a significant fraction of a population at a significant fraction of the speed of light. We’re talking about millions of people here, travelling at least at 0.1c. Even with the most powerful fuel conceivable – antimatter – it’s painful to even look at Tsiolkovsky’s equation. At this point people usually postulate that some magical technology will solve all problems. That’s perhaps good enough for a science fiction novel, but not for a scientific argument. In any case, if living space is needed, it is much easier to build Ian Banks’-style Orbitals, and if you make them smaller magical technology is not even needed.

    If your home star is actually dying that’s another story, but then the civilization will migrate once, to a longer-lived star, and stay there for a couple billion years.

    One could also just do scientific exploration, instead of colonization. In this case we might simply not see the probes, but there’s also the problem that exploration gets increasingly boring with distance. Sure, we’ll definitely send a probe to Alpha Centauri, that’s just 5 light-years away, we’ll probably hear back from it in 55 years (at 0.1c). And Sirius, the next closest star? At 9 light-years it’s already a much worse deal, taking 99 years to hear from it. We’ll probably be patient enough. But Aldebaran? 65 light-years away? Will we wait 715 years? Unlikely. Even if we have some magical technology that allows us to hear back from it in 130 years, we can’t get much further. Aldebaran is 550 light years away. Magical technology is needed to make a probe that can last 550 years, and magical patience is needed to wait 1100 years to hear back from it. For a sense of scale, the radius of the Milky Way is roughly 100,000 light-years. This is not even even leaving our neighbourhood.

  123. lewikee Says:

    My feel on this is that the ridiculously improbable steps you mention that eventually lead to your blog’s existence aren’t all that improbable given a life with a reasonable foothold on a planet. Life becoming more complex so as to adapt to a changing environment is just part of the phenomenon. It comes with the price of admission.

    The price of admission is abiogenesis. We just don’t have any good sense of its probability, even given favorable conditions. I think if were to devote more research towards fleshing out a better probabilistic model for abiogenesis – almost entirely through experimental attempts to induce it – we’d be able to actually (and finally) put Drake’s equation to good use.

    The civilization that is eventually able to propagate itself through the universe at arbitrary speeds is the one that knows the ins and outs of abiogenesis and knows how to seek out planets that are particularly likely to result in the event, for good (wanna hang out/get assimilated?) or for ill (let’s just nip this in the bud).

  124. Scott Says:

    anthropic #121: Our relative earliness is a fact that some astronomers learned in the 20th century and that most people still live their lives without knowing. As such, it’s very easy to separate from more “basic, defining” facts about the human experience, like the fact that we’re here as a technological civilization at all. It’s not nearly as contaminated by selection bias.

  125. Monday assorted hyperlinks - Marginal REVOLUTION - Forbes Best Says:

    […] 5. Scott Aaronson on Robin’s latest ideas. […]

  126. anthropic Says:

    Scott #123: Ok, but it’s still arbitrary where to draw the line between “basic” and “non-basic” information. The result of a Bayesian argument must not depend on that choice. We have to condition on all available information and then the argument collapses.

  127. Boonton Says:

    A sphere expanding at nearly the speed of light? I think this is imply some type of cosmic gentrification where the aliens are zipping as fast as possible into every solar system around, transforming/using/whatever/ that solar system and then launching off to the next one.

    1. Even being a bit off of light speed adds up pretty fast. If you are expanding at 90% of light speed, after 100 light years a gap of 10 years will open up between us being able to see the sphere from outside and the actual sphere arriving towards us.

    2. Even if they expended huge energy to be more at 99.99% of light speed rather than the more economical 90%, they have to stop when they hit a new star system and then do whatever it is they do and then launch again. This will again open up a gap between us being able to see the sphere and it actually arriving.

    3. Stars are not evenly distributed so their expansion from star to star would look less like a sphere and more like a bush with them following branches of dense star areas while stars that are more off to the side by themselves will wait longer. This too will open up the possibility to see the sphere before it actually arrives.

  128. Nathan Myers Says:

    Pascal #119: A defining fact… It is only a conspiracy if it is illegal. We’re not here to talk about tictacs, but if I spared one reader from wasting cycles on them, I am ecstatic.

    The larger point of #101, and of #57 and the final paragraph of #65, is that the Drake Equation is missing an unknown and possibly large number of factors that are, therefore, not discussed. Each factor added …. well, inserted … can only make the result smaller.

    Hansen’s contribution is our advent vs. the forecast lifetime of the universe. One way to interpret it is Hansen’s, that we are early relative to everybody else. Another is that we are dirt-common but other factors, known or unknown, don’t favor contact. As a universal principle, guessing there are things we don’t know about is pretty safe; we usually demand proof that there isn’t. Assuming there isn’t may be necessary to continued discussion, but the gruel thins.

  129. Scott Says:

    anthropic #126: Right, this is a common problem in statistics. The string theorists lament that they could’ve gotten credit for predicting general relativity, but for the “historical accident” that it already existed at the time. In the same way, Robin could’ve gotten credit for predicting our cosmic earliness from his model, but for the “historical accident” that it was already known!

  130. J Says:

    Scott #120,
    Let’s assume Hansen was right, then the proportion of intelligent beings that would live during the early area should be almost zero. From a purely logical point-of-view, isn’t that « out of the frying pan into the fire »?

  131. anthropic Says:

    Scott #129: He could have predicted our cosmic earliness if he had strong other, non-anthropic arguments for these future bubbles. But the bubbles are definitely not supported by our cosmic earliness. My argument #117 stands.

  132. Sniffnoy Says:

    Scott #46:

    I think it’s worth noting what particular form of the anthropic principle it uses; my understanding is that the argument uses SSA?

  133. Scott Says:

    J #130: The relevant question is whether the beings in the expanding bubbles even belong to the same “reference class” as we do, for purposes of anthropic reasoning. If we say that they don’t—i.e., that being “human-like” is central to what we are, that these post-singularity beings might not even have a form of consciousness comparable to ours, etc.—then Robin’s model does render it less mysterious why the universe would’ve “rushed to produce us as quickly as it could,” give or take ~6 billion years. I agree that the assumption about the reference class is a huge one, which is why I made sure to call attention to it in the original post!

  134. Scott Says:

    anthropic #131: Let’s make this more concrete. I claim that, if there turn out to be excellent non-selection-related reasons why we’re near the beginning—for example, if future astronomical discoveries show that it’s almost impossible for dwarf stars to support the evolution of life—then Robin’s model is effectively falsified. The expanding bubbles might or might not exist, but nothing would then be explained by their existence.

    Conversely, though, if future astronomical discoveries affirm that there totally could be life around dwarf stars trillions of years from now, thus heightening the mystery of our earliness, then we’d have to consider Robin’s model to have made a successful prediction, albeit only a mildly impressive one.

  135. Pascal Says:

    Nathan Myers #128: “We’re not here to talk about tictacs”… To make everyone happy (?), I would like to point out that the “tictacs are aliens” theory is compatible with the “zoo theory”, and according to Scott the zoo theory is compatible with Hanson’s analysis.

  136. J Says:

    Scott #132,
    Interesting but, you say this point was already in the original post? Sorry, I can’t find it. Would you mind to point out the relevant sentences?

  137. Barkley Rosser Says:

    Scott et al,
    I saw this at Marginal Revolution. I have not read all comments here thoroughly, but a couple get at some issues I have with this. Basically I see no reason why Robin’s (I know him well personally) three alternatives are all there are, and I find his #3 highly unlikely. Just to mention two others I think not mentioned: 1) intelligent life is out there, but highly scarce, so our fairly short SETI time just has not gotten to any of it that would be interested in communicating back to us at our level and using our technology, not to mention it may be that the speed of light really is a limit for space travel, which puts a big crimp in a lot of things, 2) the old sci fi hypothesis that indeed there is a galactic or even larger multi-system civilization that is simply far above us in intelligence and technology, that is watching us to take us in when we reach an appropriate level, and we do not observe them (aside maybe from some UFOs) because they communicate with a technology different from and far superior to what we use.

    Oh, one more point. I checked. Best estimate for when the sun will swallow up earth is more like 7.5 billion years, not just 1 billion, although it is also the fact that a really nasty solar flare could make a bad mess of things here, something that gets less attention than an asteroid hitting earth, but also something we can do less about.

    J. Barkley Rosser, Jr.

  138. Gerard Says:

    Nathan Myers #128

    > A defining fact… It is only a conspiracy if it is illegal.

    In a legal context conspiracies are indeed illegal by definition. However in the context of “conspiracy theories” illegality is not necessarily a defining characteristic. Most people would probably consider Hitler’s campaign to wipe out the Jews a conspiracy, though, as far as I know, it wasn’t illegal by relevant national laws at the time.

    > We’re not here to talk about tictacs, but if I spared one reader from wasting cycles on them, I am ecstatic.

    If your hypothesis is correct it should be seen as a major political issue. We would have not only the DoD overtly lying to the American people but also to Congress and/or involving Congress in a disinformation campaign. If the people have become complacent enough to tolerate such acts democracy will not long survive in this country.

    That’s why I think this subject is so important. No matter what the explanation turns out to be it is likely to have a major impact on how we perceive our world.

  139. Scott Says:

    J #136: Here’s what I said in the post.

      Robin admits that his account leaves a huge question open: namely, why should our experience have been a “merely human,” “pre-bubble” experience at all? If you buy that these expanding bubbles are coming, it seems likely that there will be trillions of times more sentient experiences inside them than outside. So experiences like ours would be rare and anomalous—like finding yourself at the dawn of human history, with Hammurabi et al., and realizing that almost every interesting thing that will ever happen is still to the future. So Robin simply takes as a brute fact that our experience is “earth-like” or “human-like”; he then tries to explain the other observations from that starting point.
  140. Scott Says:

    Barkley Rosser #137: To recap some of the discussion we already had upthread—sure, all the things you mention are possible, but they all still leave the challenge of explaining our anomalous earliness in the cosmic calendar (i.e., why have we shown up after a mere 13.7 billion years, rather than trillions of years?), the main empirical observation that Robin is trying to account for.

    My understanding was that the sun is going to boil away the oceans after a mere billion years? It seems clear that, if human-originated sentience is still around after that and still able to survive on the earth, it will only be because it’s also able to survive off the earth.

  141. anthropic Says:

    Scott #134: I’ m interested in the anthropic part of the argument, the one based on our earliness, and that doesn’t work in the slightest. Suppose there are two models of our universe: both completely agree about the probability of life in the neighborhood of sun-like stars. For later cosmic times, one predicts bubbles, the other doesn’t. Suppose both models seem to be equally likely a priori (i. e. before applying anthropic arguments). Now let’s weigh in our own existence and experience (early in cosmic times, near a sun-like star etc.) As both models predict the existence of observers with these experience with the same probability, we’ve learned nothing.

  142. Nathan Myers Says:

    Gerard #138: That is certain: the US *did* invade Iraq, transferring $5T+ from the public to… people. So it’s at least $5T important, but off-topic here/now.

  143. Gerard Says:

    Scott

    You’ve talked a lot about how “early” we are but you haven’t explained why we should believe that assertion and those of us less familiar with astrophysics probably don’t have a clear grasp of the argument.

    Is it based solely on the idea that the universe should produce many white or red dwarfs which should be able to support life for long periods of time ?

    I wonder if our understanding of stars and solar systems and their formation is really good enough to have high confidence in that prediction, given that we we still only have one example of a solar system that’s been extensively studied.

    Also if you’re talking about trillions of years that’s only 3 orders of magnitude from where we are. I’m sure many millions of people have won lotteries with worse odds.

  144. Raoul Ohio Says:

    Remarks:

    0. The Drake equation has zero content. The DE presents a quantity as a product of a bunch of factors, most of which nothing is known about.

    1. As Mateus Araújo and I separately pointed out above, the entire FP issue is a joke, because there is no way any such civilization could ever know about any other one, let alone go bother them.

    And this is obvious. How so many otherwise smart people confuse their scientist hat with their science fiction fan hat on this issue is a question worth study. When talking Quantum computing, you believe in physics, and when talking alien civilizations, you don’t? [Insert your own clever wisecrack about about legalization of pot here.]

    2. I have been closely following this issue since before Shklovsky + Sagan’s “Intelligent Life in the Universe”, and I can point to two major changes over 60 years in what we do know:

    2.1. The places to look for life has GREATLY EXPANDED. in 1960’s there was thought to be life on Earth and a remote chance of one time life on Mars, and surely no where else in the solar system. Now one time life on Mars is perhaps likely, current life on Mars is plausible, there are a half dozen other plausible places for life in the solar system, and a bunch more long shots. A dozen different places in the solar system with life is not out of the questions.

    2.2. The chances of any life growing to advanced civilization status has GREATLY DECREASED. The understanding of bad things that can happen has greatly expanded. Most of these would not eliminate life, but would batter it regularly.

  145. J Says:

    Scott #139 #132
    Thanks, connecting these dots helps. One remaining confusion I have is: why, under Hansen’s proposal, should we find ourselves somewhat early rather than almost maximally early?

  146. Nikny Says:

    Maybe I’m missing something here, but to me this sounds like a pretty lame attempt to model the physical world typical to an economist 🙂

    I think the scenarion 3 about “maxing out physical resources” is highly dubious, but even given it’s valid:

    1 Why a sphere?

    So if we go about reason about this by observing the exactly one empirical example we have, namely the human civilization (or perhaps the earths biosphere more broadly), maybe it seems like it’s distributed spherically, since we’re on a roughly spherical planet (duh..;) But why would it in any way what so ever logically follow that the expansion would be spherical? More likely it would be according to some other weird trajectories and geometries completely unrelated to a sphere..(unless I’m missing something physical, I’m not a physicist..)

    2 Why approaching speed of light?

    Again, simply highly dubious. But Ok, given that the civilization would expand in some exponential way that would get it up to some speed like that after some time, it should be possible to estimate how long time this would take by observing the one sample we have. I admittedly did not take the time to carry out an actual calculation given some reasonable model assumptions, but I feel pretty confident that by taking into account the speed at which the “biospheric civilization” has “expanded” over time so far, the exponent of the “spherical expansion” would not be noticeable.

    Maybe someone here can give better answers to my questions 1 and 2 with regards to the proposal by Hanson?

  147. Peter Erwin Says:

    Scott, I’m not sure I understand why you think “our position relative to the lifetime of all the stars makes it look as though we arrived crazily, freakishly early”. An argument could be made that we are relatively late, since the peak of the star formation rate in the universe was about 10 billion years ago, and the Sun formed about 5.5 billion years after that (roughly speaking, about 80% of the stars that have formed up until now formed before the Sun).

    I get the impression you think the chances of abiogenesis are independent of time (“a Poisson process”), and so a red dwarf could have life on its planets forming (etc.) 100 billion years after it formed just as easily as 1 billion years after it formed. But this is rather unlikely, since it assumes that planets don’t evolve, whereas we know they do. For example, they cool down as they age, which means less volcanic activity. Atmospheres and hydrospheres evolve, too. There’s good evidence Mars was originally a bit warmer and wetter, probably with a denser atmosphere — but now it’s cold, dry, and has a fairly minimal atmosphere. Venus plausibly had an ocean early on, but it evaporated as part of the runaway greenhouse effect. Even if you could somehow tamp down and magically prolong the Sun’s evolution, so it stayed the same brightness for the next trillion years, it’s highly unlikely you’d get abiogenesis on Venus or Mars any time in the future.

    I think it’s more likely that there’s a relatively limited window after planetary formation for the conditions favorable to abiogenesis to persist, so it’s probably not the case that chances of life appearing one or two hundred billion years after a red dwarf’s formation are the same as the chances of it appearing in the first few hundred million years.

  148. Phil Jensen Says:

    I fail to understand why you think we arrived “crazily, freakishly early”. Once enough star cycles have happened that there are planets with a mix of elements like Earth’s, what is the impediment to life?

    Our recent experience is overwhelmingly conditioned by the fact that Earth was cursed with a large (but, obviously, finite) stock of reduced carbon (coal and hydrocarbons). We have priced this essentially at the cost of extraction, which amounts to a decision to throw a big, incredibly wasteful party for a few centuries, and damn the hangover. Extinction is staring us in the face, for those with eyes to see.

    An intelligent species that found itself on a planet lacking such “fossil fuels” would take a much different course.

  149. Bruce Smith Says:

    Scott #139: “… like finding yourself at the dawn of human history, with Hammurabi et al., and realizing that almost every interesting thing that will ever happen is still to the future. …”

    I think that situation was not obvious to most people at the time — just like the corresponding situation is not obvious now. But some of us are optimistic, and think that with nonzero probability, that statement will be true at *all* times!

    Others #?: If you like to think about very different kinds of humans, and/or beings using very different planning timescales, you would probably like John C. Wright’s SF series that starts with “Count to a Trillion” and ends with “Count to Infinity”.

  150. Peter Erwin Says:

    Barkley Rosser #137 and Scott #140:

    There’s a good summary of some of the issues in the Wikipedia article for Future of Earth.

    Basically, you have several nasty things happening in succession:

    1. As the Sun gets brighter, the Earth’s surface will get warmer. This will lead to increased weathering, which will draw CO2 out of the atmosphere faster (some of this will be replenished by vulcanism, but the amount of the latter is slowly decreasing as the Earth’s interior cools). After approximately 600 to 800 milllion years, the CO2 level in the atmosphere will become too low for most plant life.

    2. As the plant life diminishes and dies off, oxygen levels in the atmosphere will fall (as will ozone levels in the stratosphere), so that animal life will start dying as well (those animals that survived the loss of plants, if any).

    3. As the Sun continues to increase in brightness, the oceans will evaporate to the point where you get may get a “moist greenhouse”, which increases the surface temperature and thus the evaporation rate. By 1 or 1.1 billion years from now, it is likely that most of the oceans will have evaporated (the water in the atmosphere will subsequently be lost via UV light from the Sun splitting the molecules into H and O, with the H escaping from the atmosphere).

  151. Scott Says:

    anthropic #141:

      Suppose there are two models of our universe: both completely agree about the probability of life in the neighborhood of sun-like stars. For later cosmic times, one predicts bubbles, the other doesn’t. Suppose both models seem to be equally likely a priori (i. e. before applying anthropic arguments). Now let’s weigh in our own existence and experience (early in cosmic times, near a sun-like star etc.) As both models predict the existence of observers with these experience with the same probability, we’ve learned nothing.

    What you say would be true under what Nick Bostrom calls the “Self-Indication Assumption” (SIA), according to which (all else equal) you’re twice as likely to exist at all in a world with twice as many observers, so you have the same likelihood of being some particular observer. But it’s false under what Bostrom calls the “Self-Sampling Assumption” (SSA), according to which your own existence is not probabilistic evidence for observers being common: it’s just, first a world is chosen randomly, and then you’re chosen randomly from that world (if it has any observers at all). Under SSA, if intelligent life could arise at random times throughout the universe’s first trillion years, then we’d expect to be later and should indeed be surprised to find ourselves so early.

    SIA and SSA can both be made to yield paradoxical, insane-sounding consequences—please read Bostrom’s book for more—indeed, I’m not sure that one has a clear advantage over the other in that department. Certainly, though, if you think we shouldn’t be surprised to be among the earliest observers, then turning the same Bayesian crank, the more observers there are in some future scenario, the likelier you should consider that scenario to be (all else equal). Are you OK with that consequence?

  152. Scott Says:

    Gerard #143: Yes, the argument for earliness is based on the fact—which I hadn’t appreciated myself until recently—that dwarf stars will continue to shine (and new dwarf stars will even be formed) for trillions more years. Of course, it’s possible that this is irrelevant, since the evolution of complex life requires a sun-like star for some reason that’s not yet fully known. But if life could evolve just fine around a dwarf star, then there’s indeed a puzzle about why we’re so early (in relative terms), and any scenario that can explain it wins points compared to scenarios that treat it as just a lucky accident.

  153. Scott Says:

    Raoul Ohio #144:

      the entire FP issue is a joke, because there is no way any such civilization could ever know about any other one, let alone go bother them.

      And this is obvious. How so many otherwise smart people confuse their scientist hat with their science fiction fan hat on this issue is a question worth study. When talking Quantum computing, you believe in physics, and when talking alien civilizations, you don’t?

    You’re wrong. None of this stuff is obvious. Maybe we’re alone in the whole cosmos. Maybe primitive life is common but intelligent life is rare. Maybe intelligent life commonly arises but quickly wipes itself out. Maybe intelligent life is common and we’d find it in a decade if we made a real effort to look. Maybe we’ll only find out about it when it arrives on our doorstep. If you polled the greatest scientists of the past century, they’d be all over the map in their leanings on these questions. Almost none would tell you that the answers were obvious. If we’re lucky, we can clarify the space of possibilities just a tiny bit with a new observation here, a new theoretical argument there.

    You know, Robin often gets accused of building towers of “plausible extrapolation” on balsawood foundations of fact, like some cartoon stereotype of an economist. So it’s ironic that I—the one explaining his scenario—am also the one who has to remind people of the crushing weight of uncertainty whenever we theorize about extraterrestrial life! 🙂

  154. Scott Says:

    J #145:

      One remaining confusion I have is: why, under Hansen’s proposal, should we find ourselves somewhat early rather than almost maximally early?

    Under all the stated assumptions—e.g., that civilizations will form these bubbles that take over the observable universe within the next 10-20 billion years; that we were chosen “randomly” but only from among the pre-bubble civilizations—we’d expect to find ourselves at a more-or-less random time within the first 20-30 billion years after the Big Bang, not seeing any of the other civilizations yet. FWIW, this matches our observations.

  155. Scott Says:

    Nikny #146:

      1 Why a sphere?

    Because, um, if you’re trying to take over resources as quickly as the laws of physics allow, then the limit is your whole future light-cone? (Having said that, not much would change if for some reason civilizations expanded, say, 10% faster in some directions than in others—except, of course, that in the slower directions they’d necessarily be expanding at most at ~91% of the speed of light, so there’d be more time to detect them.)

      2 Why approaching speed of light?

    The obvious reasons are (1) to take over resources as quickly as possible before anyone else does, and (2) even more importantly, to prevent anyone else from having advance warning that you’re on the way.

    To clarify, it might well be that 99.9% of civilizations are not aggressively expasionist and don’t think this way at all! In that case, though, the ones we might someday see taking over huge swathes of the night sky would (alas) be other 0.1%.

  156. JimV Says:

    Probably some people have already stated this opinion, but there were 149 comments when I got here, too many to check, sorry.

    The idea of a space-faring civilization expanding its bubble at nearly the speed of light is not consistent with the laws of the universe as we understand them, it seems to me. Anything that arrives here at near the speed of light is going to go zipping by at that speed. To get from A to B stopping at B you have to first accelerate to the midpoint and then decelerate the rest of the way. Even neglecting relativistic effects the average velocity in our rest frame would be less than the speed of light, and probably a lot less..

    Then there’s the energy needed to do this, versus other things you could use that energy for at home, and the danger of colliding with an ejected planetoid. Getting from here to Proxima Centauri would require a huge effort with a huge chance of never returning. I for one would not sign up for that mission. Let the robots do it, but unless we force them, why would they want to? They don’t need earth-like planets.

    For the same energy and effort expenditure we could probably build self-contained habitats that float in empty space. That’s what I would do. I’d probably make them so they block transmission of internal EM also, for paranoia’s sake. I might let them drift toward the outskirts of other solar systems for more raw materials, or I might not if I could hose up enough gas to to sustain my fusion reactors. I wouldn’t need much, because we would reproduce as AI entities. No, not downloaded from human brains, but AI children developed to emulate the best of human cognition, raised as children by the last generations of biological humans, then raised by the previous AI generations.

    Okay, we won’t survive long enough to make that happen, but it makes more sense to me than the Hansen scenario. Things like Trump and climate-change and pollution explain the Fermi paradox adequately for me. The rest is science-fiction. (Some great, like “A Deepness in the Sky”, most not so great.)

    In any case, EM radiated signals from another space-faring, expanding civilization ought to reach us long before actual spaceships if such were even feasible, by the above argument.

  157. J Says:

    Scott #154
    Yes it’s consistent with what we see, but isn’t that’s because Hanson’s reasoning treat the time of our apparence as a given assumption? A more interesting prediction, I think, is that we should expect that the next bubble we’ll meet will come from within 0.25-1 G light year, e.g. from outside our galaxy. I’m afraid this also leave us to explain why we should be the first within this seemingly rich Milky Way.

    JimV #155
    One non agressive exemple of the tasks one would want to achieve through this (most likely robotic) expansion is long-distance entanglement (we use that already in some of our telescopes).

  158. Scott Says:

    J #157: I would say it differently. Robin tells a story that makes no special reference to earth or humans—one where first there’s a Big Bang, then stars form, then life and civilizations form around the stars, then there’s a “competition of great powers” era as machines or lifeforms descended from those civilizations scramble to repurpose all the raw material in the universe to their own ends, and then by (let’s say) 20 or 30 billion years after the Big Bang the competition is over and the whole universe is transformed. He then asks what a random civilization just becoming technological would expect to see in such a universe and the answer is: what we see now! A “mere” 10-20 billion years of history and no other civilizations yet visible.

    Granted, there are probably many other ways to solve the “inverse problem” that Robin set himself — and as long as that remains the case, we’ll lack strong evidence that Robin’s scenario is actually true. But crucially, most of the alternatives that I’ve seen discussed don’t even solve the inverse problem, of accounting for what we see! My hope is that having some candidate solution in front of us could help move the discussion forward.

  159. duck_master Says:

    Wait, has anyone considered the possibility that we’re in an expansionist civilization’s bubble right now but we just haven’t noticed it? (If any expansionist civilizations protectively enclose developing ones in spherical “containers”, which seems likely, then this situation is bound to happen many times over.)

    On another note, I concur with Robert GT #103 and Greg #105 that the “Dissolving the Fermi Paradox” paper has been the best treatment of the Fermi Paradox that I’ve seen so far. However, I respect Robin Hanson for his idea. (And the proposal of Rollo Burgess #109, Scott #115 is just a more specific version of the idea that outcomes #1 and #2, in Hanson’s taxonomy of civilizational fates, is extremely common!)

    On a third note, the Bayesian arguments on the Fermi paradox may turn out very differently depending on whether we adopt the Self-Indication Assumption or Self-Sampling Assumption. (I currently weakly prefer Self-Indication, but I could go either way.) This needs more attention.

  160. J Says:

    Scott #158
    Actually Greg Egan sketched two other candidate solutions that would as well « explain » why we’re early (same comment thread as Vargas):
    – maybe every society smart enough to bubble don’t seek maximal material growth and starts sharing this wisdom, in which case the very fact that we are stupid means we’re early
    – maybe every society smart enough to bubble can create anything at will and can’t possibly learn anything from elsewhere, in which case the very fact that our technology is primitive means we’re early

    (Egan did not make the last part of these arguments and may or may not disagree with this twist)

  161. Raoul Ohio Says:

    I have no clue about how common (1) life and/or (2) intelligent civilizations are in the galaxy/universe. My guess is (1) everywhere (say, around half the stars) / (2) quite a few at any time per big galaxy, but that is a pure guess.

    My whole point is that for sure there will never be interstellar travel anywhere, and unlikely to ever be interstellar communication.

    “Spheres of civilization increasing at near light speed”? Sure — I’ll have two of whatever they have been drinking.

    Let’s stick to to the basics, like can Mothra beat Godzilla?

  162. Scott Says:

    Raoul Ohio #161: I don’t know where your confidence comes from. How do you know?
    What if there are creatures who live millions of years, for whom traveling to a nearby star (even at a small fraction of c) might seem perfectly reasonable? What if there are creatures in the habit of sending frozen embryos to millions of other stars, hoping someone will pick one up? It’s a big universe, and you’ve already guessed that it has many civilizations, who we can presume will try many things, limited by the laws of physics but not by our imaginations here on earth.

  163. Pascal Says:

    Scott #162 Raoul Ohio #161: I don’t know where your confidence comes from. How do you know?

    I’m with Scott here. Moreover, are we really sure than that faster than light travel is impossible?
    Yes, it’s impossible according the known laws of physics. But considering how much our understanding of the universe has changed in the last few centuries, it would be a little presumptuous to assert that it will remain essentially identical in the next 200 years or the next million years.
    And we may not have to wait for a million years.
    I could point you to at least 2 very different proposals for FTL travel that only extrapolate a little bit on current physics.

  164. Mateus Araújo Says:

    Raoul Ohio #144: That neither what I said nor what I meant. I just find the idea of expanding through the galaxy ludicrous (let alone a near lightspeed!). I don’t find the Fermi paradox a joke, and I find perfectly plausible that two civilizations would meet each other. Not only plausible, but I find very probable that at least once it happened that two civilizations evolved relatively nearby and more-or-less at the same time, so that the could actually communicate.

    But that’s it. Suppose there was another civilization at Alpha Centauri. We’d definitely be interested in talking to them, and doing trade on information (takes about 10 years to finish a transaction but that’s ok, the immense value of the information easily generates enough patience). But doing trade on goods? Can anyone conceive of a good that would be cheaper to order from 5 light-years away than to produce locally? I can’t.

    And how about visiting them? Extremely difficult and expensive, but I think we would eventually do it just for the hell of it. But routine travels between the Sun and Alpha Centauri? Pointless, insane. And making war? That’s just plain ridiculous. With a lot of difficulty we could send a small ship that would take a couple of decades to get there, and then need to fight the entire bloody civilization.

    I find it entirely unsurprising that Robin Hanson is an economist, not a physicist. If the only thing you take from physics is the lightspeed limit, sure, interstellar war might sound conceivable. But if you start to actually calculate the time, mass, energy needed with any remotely plausible technology, the answer is lol.

  165. Scott Says:

    Mateus #164: Robin did his undergrad degree in physics before switching to economics. Dyson, Hawking, Sagan, Kardashev were just a few of the well-known physicists and astronomers to write about interstellar civilizations, so I really don’t think that’s the dividing line here.

  166. Mateus Araújo Says:

    Scott #165: Ok, that was an ad hominem, and can be answered by an ab auctoritate.

    I can turn it into an argument about content, though: I did read Hanson’s blog posts, and there’s zero physics there. He didn’t say anything about the difficulty of interstellar travel, and gave no argument to support his assumption of aggressively expanding civilizations. Until he does write something about it I can safely discard everything.

  167. lewikee Says:

    Does anyone know this: Let’s say there was another civilization with our current technological capabilities about 20K light years away from us (roughly 1/4 of galaxy away) that did whatever we’re doing now 20K years ago. Would they “come up on our screens” right now given our current methods? If not, would we be able to find them by quadrupling our funding in those methods?

    I am trying to get a sense of how easy/hard it actually is to find other civilizations so as to judge the claim made by the Fermi paradox.

  168. Gerard Says:

    Pascal #163

    The fundamental problem with any proposed method of FTL is that they all result in causality becoming relative. Since all of science and human reason is built on causality that would result in such a radical transformation of our understanding of the world that we would practically have to start over from scratch.

  169. Daniel Kokotajlo Says:

    The simulation hypothesis is the answer to this puzzle btw:

    “Robin admits that his account leaves a huge question open: namely, why should our experience have been a “merely human,” “pre-bubble” experience at all? If you buy that these expanding bubbles are coming, it seems likely that there will be trillions of times more sentient experiences inside them than outside. So experiences like ours would be rare and anomalous—like finding yourself at the dawn of human history, with Hammurabi et al., and realizing that almost every interesting thing that will ever happen is still to the future. So Robin simply takes as a brute fact that our experience is “earth-like” or “human-like”; he then tries to explain the other observations from that starting point.”

    If you want to know more, Scott, just send me an email. Hanson’s doing great work by making all of this formal and published, but the informal version has been circulating for years.

  170. anthropic Says:

    Scott #151:

    “Certainly, though, if you think we shouldn’t be surprised to be among the earliest observers, then turning the same Bayesian crank, the more observers there are in some future scenario, the likelier you should consider that scenario to be (all else equal). Are you OK with that consequence?”

    No, because future observers living near white dwarfs will definitely make different observations than I do. Therefore increasing their number doesn’t increase the probability of my experience. A higher number of present epoch observers near sun-like stars does.

  171. Nikny Says:

    Scott #155

    I still find the sphere thing (let a alone the speed of light thing) very unconvincing. Obviously this whole discussion is pure speculation, but it’s nice to at least try to ground it in something. Could you please expand a bit on how exactly you (or Hanson) imagine this hypothetical mechanism by which this “civilization” propagates?

    I’m thinking something like the paper-clip transformer, maybe some AI implemented in some planc-scale nano technology running amoc, or something like that. But still, why anticipate that this mechanism, which is obviously entirely unlike anything we have previously seen, theoretically stipulated, or even convincingly imagined, would propagate uniformly in every direction? Isn’t it more likely it would say leap between planets/galaxies in some weird non-uniform pattern, or maybe it would propagate in some chaotic non-local way according to some yet unknown physics? Or at least I find that not less convincing that it would be a nice-to-imagine type of shape like a sphere..

    And also still have a hard time imagining that the light-speed propagation of whatever mechanism like this wouldn’t be ruled out by serious time/energy-restrictions.

  172. General Says:

    How can we be early when there was complex life on earth more than 100 million years ago? I’m not aware of any reason dinos/sharks couldn’t have become intelligent, etc., so you can’t assume we’re somehow the “right” timing for intelligent, technological life. And that 100 million years matters, obviously, not just a rounding error in terms of how much progress you could make in that time (in terms of exploration, tech, etc.).

  173. J Says:

    Mateus #164
    https://en.wikipedia.org/wiki/Self-replicating_spacecraft#Von_Neumann_probes

  174. J Says:

    lewikee #167
    Nope. Details here: https://iopscience.iop.org/article/10.3847/1538-3881/aae099/pdf

  175. Kailer Mullet Says:

    What if we do see giant alien civilization. Maybe that’s what all that dark matter is. Some alien civilization consuming all the light energy.

  176. Scott Says:

    lewikee #167:

      Does anyone know this: Let’s say there was another civilization with our current technological capabilities about 20K light years away from us (roughly 1/4 of galaxy away) that did whatever we’re doing now 20K years ago. Would they “come up on our screens” right now given our current methods? If not, would we be able to find them by quadrupling our funding in those methods?

    Your question is an excellent one! I was at a talk by Jill Tarter (said to be the inspiration for Ellie Arroway in Carl Sagan’s Contact), where she said that what SETI has done so far is roughly analogous to looking for fish in a single cupful of water from the ocean. On the other hand, Yuri Milner’s $100 million “Breakthrough Listen” project is soon going to monitor probably more channels than all previous SETI searches combined, apparently including the million stars closest to earth. But I believe there’s still the problem that, while it could easily find signals directly aimed at us, finding signals aimed at no one in particular is harder. I’m not even sure if we’d detect exact replicas of earth! Does anyone who knows more want to comment?

  177. Scott Says:

    anthropic #170:

      No, because future observers living near white dwarfs will definitely make different observations than I do. Therefore increasing their number doesn’t increase the probability of my experience. A higher number of present epoch observers near sun-like stars does.

    See, this is what makes anthropic reasoning so tricky! I have some commenters in this thread who think that post-singularity nano-entities spreading across the universe at nearly the speed of light need to be included in our reference class. And then there’s you, who wouldn’t even include hominids on a planet that happened to orbit a white dwarf instead of a sun-like star!

    If you draw your circle, of “beings for whom you wonder why you weren’t them,” so tightly that it only includes beings with your exact same observations, then doesn’t the anthropic principle become basically useless, except for universes so enormous that we’d expect them to contain exact replicas of you?

  178. Scott Says:

    Nikny #171:

      I still find the sphere thing (let a alone the speed of light thing) very unconvincing. Obviously this whole discussion is pure speculation, but it’s nice to at least try to ground it in something. Could you please expand a bit on how exactly you (or Hanson) imagine this hypothetical mechanism by which this “civilization” propagates?

    To repeat what I said in comment #47:

      Actually, if I were writing this as a sci-fi story, I’d probably make the “expanding sphere of civilization” literally a black hole! (Surrounded by self-replicating nanomachines that grab everything in sight and throw it into the black hole in a very precisely choreographed way.) This black hole would support “sentient life”—in fact, the most sentient life that it’s possible to support in any spatial region of the same size—by doing Planck-scale computations in the quantum-gravitational degrees of freedom on its event horizon.
  179. Pascal Says:

    Gerard #158: there is the “warp drive” proposal, which sounds like science fiction but is apparently serious enough to be discussed recently by Sabine Hossenfelder on her blog. She does point out some problems with this idea, but no causality problem if I remember correctly.

    Another much more precise proposal is the Janus cosmological model by Jean-Pierre Petit (https://januscosmologicalmodel.com/). In this model superfast travel model is possible if we manage to jump into the “twin universe” attached to our own (these can be viewed roughly as the 2 sides of a paper sheet). In the twin universe the speed of light is faster than in our own, and distances are shorter. So, when the traveler emerges from the twin universe to come back to our own, the result can effectively looks like FTL travel.

  180. Scott Says:

    General #172:

      How can we be early when there was complex life on earth more than 100 million years ago?

    Yes, as far as we know, we could have been hundreds of millions or even billions of years earlier. But we also could have been trillions of years later. That’s the asymmetry that Robin is trying to explain.

  181. Scott Says:

    Kailer Mullet #175:

      What if we do see giant alien civilization. Maybe that’s what all that dark matter is. Some alien civilization consuming all the light energy.

    Interesting theory but I believe ruled out, since in the empirically-supported ΛCDM model of cosmology, dark matter isn’t “consuming” any light energy (indeed, it would be a hell of a lot easier to detect if it was!). Instead, dark matter has been gravitationally shaping the formation of galaxies since the extremely early universe.

  182. Scott Says:

    Everyone: Sorry, but I believe I’m going to bow out of this thread now — I want to see if I can do some actual work this week! But I’ll still read, to check whether anyone here manages to get to the bottom of this extraterrestrial life question. 🙂

  183. Laurence Cox Says:

    I’ve come a bit late to this, so here are some general comments:
    Gautam Goel #33
    The first appearance of this idea as far as I know is Michael J Hart in Quarterly Journal of the Royal Astronomical Society “An Explanation for the Absence of Extraterrestrials on Earth” Vol 16, p128ff (1975). You can also find the paper in “The Quest for Extraterrestrial Life – A Book of Readings” by Donald Goldsmith (1980). He was only assuming a maximum speed of 0.1c, but still came to the conclusion that such a civilisation could fill the galaxy in about 650,000 years.

    Bram Cohen #53
    The early universe was 75% hydrogen, 25% helium-4, about 0.01% deuterium and helium-3, and about 1 part in 10¹⁰ lithium-7. The metals (elements heavier than lithium) were created in stars, but in supernovae, not novae. That’s fairly well understood for elements up to the iron group, but it is quite difficult to make the very heavy elements, and the latest thinking is that they may be formed in neutron star mergers. Our sun won’t go nova but will lose much of its mass in the red giant phase. Look at ‘star in a box’ from the Las Cumbres Observatory https://starinabox.lco.global/#

    Although we divide stars into Population I (like our sun) and Population II (globular cluster stars) older Population I stars have lower metallicity (which means any planets formed around them will be small (think asteroid-sized). I have seen a graph of metallicity against star age (to my regret I didn’t think to copy the link at the time) which showed our Sun as having a high metallicity for its age. This may be an important factor.

    Michael Weissman #73
    We know that life got started pretty soon after the Earth formed (late heavy bombardment ended about 3.8 billion years ago, earliest indications of life 3.5 billion years ago). The big step seems to be the origin of Eukaryotes (cells with a nucleus) which didn’t appear until between 1.5 and 2 billion years ago. As the Great Oxidation Event was about 2.4 billion years ago, it clearly took a long time for cells to take advantage of free oxygen in the atmosphere.

    Nick Nolan #100 See Fred Hoyle’s science fiction book “The Black Cloud” (1957). The cloud is discovered because they notice a dimming in a group of stars. It proves to be intelligent, but at first doesn’t believe that intelligent life can evolve on the hostile surface of a planet.

    arch1 #106 Tegmark is plagarising Fred Hoyle, this time “A for Andromeda” (1961) a TV series written with John Elliot. A second series the following year was called “The Andromeda Breakthrough”.

  184. anthropic Says:

    Scott #177: I don’t think so. If Theory A predicts an observer with exactly my observations to exist with probabiliy ε and Theory B predicts that with probability 2ε, then (all else being equal) I should prefer B over A even if ε is extremely small.

  185. arch1 Says:

    Laurence #183: Thanks, I didn’t know that about “A for Andromeda.” You remind me that one day an Astronomy major brought Hoyle to lunch at our undergrad student house. Despite being aware of Hoyle’s work and having read “The Black Cloud,” I was too dumb or cool or whatever to sit at his table. Live and learn:-)

  186. Isaac Duarte Says:

    With just one data point (Earth), it is really hard to accept that there are other life forms that would be so similar from us to the point of thinking about expansion, communication and galaxy colonization the way we do.

    Let’s start with the simpler forms of life. We are DNA based beings ruled by some processes that dictates the survival of the fittest and extreme competition for nutrients. What would be the consequences of not being DNA based, but something entirely different and with different purposes? And if life in other planets was focused on cooperation (instead of competition)? The concept of survival could be moot if you have immortal beings. Instead of leaving offspring with your genetic material, if you could lend memories to them? If mobility was rare or unnecessary, given that we even have immobile beings (trees) and others locked-up in some specific biome (like dolphins)?

    I know that some, if not all of these questions were answered by science fiction, even the (usually) anthropomorphic Star Trek show had, but they could certainly shed more light on the answers we seek than just extrapolating our civilization and coming up with an “Oh, they just destroyed themselves with nuclear weapons… They were just stupid, as we are”

  187. s0ph1a Says:

    The Technologically-Advanced Ape Fermi Paradox:

    As we’ve become more advanced, we’ve invented a number of attention-grabbing metal-based gadgets. Using those gadgets, we colonized every inhabitable corner of our planet and even into space. It’s only a matter of time before we colonize space. If intelligent aliens were out there, they would have invented metal gadgets too, which we would be able to see with our telescopes. If they were really advanced, they’d have spread exponentially across space, and we’d definitely be able to see them. Since we see neither of these things, we conclude that there are no intelligent aliens. What a mystery!

    The Networked-Intelligence Microbe Fermi Paradox

    When we came into our own as an intelligent, we did what any sensible intelligent species would do: We converted the upper layers of the gas giant we call home into energy-trapping nanomachines to harvest sunlight and converted the lower layers into a single undivided mass of our own bodies at the highest density possible in order to maximize our internal communications. Our energy-trapping nanomachines have a very distinctive signature, and we’ve scoured the universe for other gas giants sharing that signature. If there were any intelligent aliens out there, surely they would have converted their gas giant into a giant energy trap. Since we don’t see this, we conclude that there are no intelligent aliens. What a mystery!

    The Forest of Super-Advanced Trees Fermi Paradox

    When our species attained technological enlightenment two billion years ago, the natural thing to do was to surround our star with a flawless sphere of inward-facing mirrors to prevent the loss of heat or light to the depths of space. Our probes, stationed outside the sphere, have mapped the visible stars in the universe. Simultaneously, our ultra-precise gravity sensors have mapped the locations of all mass in the universe. If there were intelligent aliens, they would surely have done as we did and surrounded their star with a spherical, inward-facing mirror, leaving them undetectable by light sensor yet remaining visible to our gravity-sensor. However, we have detected no star-like gravity wells that do not correspond to light signatures. Since we don’t see this, we conclude that there are no intelligent aliens. What a mystery!

    The Lonely Sun Fermi Paradox

    I am the only thinking being that has ever existed. There is just me, and my light, which reaches out into the expanse of space. How do I know this? Once, I looked at the twinkling stars in the black expanse of night and wondered if any were thinking like me. So I twinkled back, and encoded messages in my twinkling. For a million million years I sent every kind of message to every other star I could see, until finally I gave up. If other stars could think, they would respond to my messages. Since I don’t see this, I conclude that there are no intelligent aliens. What a mystery!

    –from https://sapphiresonstrings.tumblr.com/post/619747068820504576/

  188. John Baez Says:

    I hope our civilization gets its act together, learns how to manage one planet sustainably, and then spreads to others. Interstellar travel under the constraints of existing human biology seems like a pointless, stupid feat: sort of like taking a goldfish to the top of Mt. Everest. So besides waiting until we get a bit wiser, we should probably wait until we develop cyborgs or robots who don’t need air and water. But once we get these basics straightened out, it could be really cool to colonize the solar systems of red dwarf stars.

    80% of the stars in the Milky Way are red dwarfs, even though none are visible to the naked eye. 20 of the 30 nearest stars are red dwarfs. Proxima Centauri, the star closest to the Sun, is a red dwarf with at least two planets!

    But the really exciting thing is that red dwarfs live a long time. They’re small — and the smaller a star is, the longer it lives. Calculations show that a red dwarf one-tenth the mass of our Sun should last for 10 trillion years! If some of our descendants want to settle down and think about things for a long time, with a decent source of power, this could be great. (Other more peppy descendants can keep spreading, and try to make sure our expanding bubble of influence survives collisions with other such bubbles.)

    One difficulty for life arising near red dwarfs is that they tend to be “flare stars”. Convection stirs up their whole interior, unlike the Sun. Convection of charged plasma makes strong magnetic fields. Magnetic fields get tied in knots, and the energy gets released through enormous flares! They can become as large as the star itself, and get so hot that they radiate lots of X-rays. Furthermore, the habitable zone for water-based life is very small for red dwarfs: for example, Proxima Centauri is just 0.17% as bright as the Sun when it’s not acting up. So, for water-based life, a red dwarf is like a very weak oven that you need to huddle close to in the chill of night — an oven that occasionally blows up in your face.

    But this need not be a deal-breaker for an advanced civilization. Furthermore, as red dwarfs age, they calm down.

    One could get a lot of thinking done in 10 trillion years.

  189. Raoul Ohio Says:

    Scott #162,

    OK, I admit I was over confident in not having thought of the possibility of things living for millions of years.

    I also did not think of things that live forever, or, for that matter, primordial beings created in the big bang that live forever. Maybe the reason no one can detect dark matter is because it consists of intelligent beings that are smart enough to defeat any measurements. Maybe it was not a big bang, but a “big bounce”, and they were here “last time”, and figured out how to ride out the singularity –that would be fun! Maybe right now they are hanging out at Area 51, and posing as “Q”. Who knows?

    Any math proof must have some axioms to start at. Likewise, in debating the Fermi Paradox,
    you need some agreed upon plausible physics. Maybe intelligent beings that live millions of years and fire off frozen embryos like Trump firing off insane tweets is plausible. But if it is unlikely, the Fermi Paradox is gone.

  190. Tamás V Says:

    Let’s not underestimate civilizations that are billions of years older than ours. They might be offended if they heard we want to tell them, based on our current knowledge, how fast they can expand… Isn’t the “expansion at nearly the speed of light” overly pessimistic? What if they find and realize some trick like the Alcubierre drive?

  191. OhMyGoodness Says:

    Baby steps-
    Apparently Raptor Engine 2 didn’t reignite and so bad landing after otherwise perfect flight of SpaceX’s Starship No.9 yesterday. I knew a guy whose wife flew on the Shuttle and she took a job in DC to preparing federal regulations for private space flight. Private space flight has an advantage over NASA in that they are not as suffocated by the federal bureaucracy as NASA. That is not acceptable for our federal bureaucrats so have to prepare another million pages or so to drown the spaceflight sector.

    We are looking to the stars but the bureaucracy is looking to increase the federal register by a plump amount. SpaceX apparently intends to drill it’s own natural gas wells to provide it’s own methane fuel. I imagine that will be a permitting nightmare even though an extremely good idea.

  192. Georgios Stamoulis Says:

    Hi Scott, all,

    I am not sure if this came up recently but I would like to draw your attention to a very important issue: University of Leicester wants to make all of a sudden all TCS jobs obsolete!
    Read the full story and sign the petition if you like in the following link:

    https://www.ipetitions.com/petition/foco-is-not-redundant

    Needless to say how absurd (even borderline illegal) this is and what a dangerous precedence will create.

    Thank you.
    Georgios

  193. Scott Says:

    Georgios #192: Thanks, I just signed it! (Even though I expect that, if the administrators cared about the views of the wider academic community, they wouldn’t be taking an axe to basic research to begin with.)

  194. Hanson, Robin Says:

    Our full paper is now out: https://arxiv.org/abs/2102.01522

    We do NOT assume aliens can expand at near the speed of light. We instead assume they expand at some speed, and then INFER a fast speed from the fact that we don’t now see them.

    We do NOT claim that SETI will fail, or that it is puzzling that it has not yet succeeded. We allow for many non-grabby aliens, but focus our attention on grabby ones. Who we agree might be very rare, but even so have outsized influence.

    I get that it can be fun to engage exotic anthropic arguments, but our argument doesn’t depend on them. They data we use are: (a) our early date of 13.8Gyr, (b) the fact that we don’t now see alien volumes in the sky, (c) the timing of events in Earth evolutionary history.

    To escape our conclusions, you need to assume very few hard steps in evolution’s history, and that longer lifetime planets are just not habitable.

  195. Justin Says:

    Ok, I wasn’t going to but here I go to Crazytown. Our ‘universe’ is some quantum field/fields and we (or maybe just me) are just some simple polynomial time verifiers. These fields share a large amount of entanglement and is/are capable of convincing us of almost anything if we play the game correctly. That’s why we are ‘early’, because all it had to do was come up with this simplish interactive protocol that convinces the verifier part that an objective reality actually exists. So the reason we haven’t seen any aliens yet is because were not playing the game properly. Find the right questions and it will convince you otherwise, thereby bringing them and their history into existence. There we go, that must be it. Do I win a prize?

  196. Overcoming Bias : An Adventure Says:

    […] More posts of mine, and one by Scott […]

  197. yenwoda Says:

    I think the exponential / speed-of-light growth is questionable. It might be that galactic civilizations expand in radius at a constant rate or even decaying rate: the bigger they get, the longer it takes information and technology to disseminate across the civilization. So each time they “level up” and double their radius, it takes twice as long to reach the next step.

  198. TDW Says:

    @Pascal–The Navy said the Tic-Tacs were unidentified, i.e. the Navy did not know what they were. They made *no* claims or implications about origin. But be aware that modern black contracting ensures the answer is unknowable. In other words, the Navy may be lying, or may genuinely believe they’re telling the truth only some other entity (another department, a skunk works…) knows better. There’s essentially no way to find out which is the case.

    In terms of “Fire Upon the Deep”–the point of that rather excellent book is that the laws of physics work differently based on your position in the universe. The “Slow Zone” caps travel at just under the speed of light and cripples the kind of exotic quantum behavior needed for super-systems and general AI. As you get further out from this Zone, the restrictions relax. As a result, most civilizations in the Zone never leave it, dying out before they can reach von Neumann-style expansion or even communicate with neighbors, but the upper zones are full of civilizations, with the ones at the very top called “gods.” I’m summarizing, but this is a great resolution of the Fermi Paradox.

  199. J Says:

    Hanson #194
    Thanks for these clarifications. Do you agree that your argument helps explaining why we’re early, or would you say we can’t conclude that as it is an input (point a)? Second, can you clarify why you consider that a-b are not anthropic arguments?

  200. anthropic Says:

    Two arguments:

    (A) I’m early in cosmic time. This increases the probability that later observers witness alien bubbles. For otherwise I could have been one of them and that would make my earliness freakish.

    (B) My name comes early in alphabetical lists. This increases the probability that people whose names come late witness alien UFOs invisible to me. For otherwise I could have been one of them and that would make my position at the top of the alphabet freakish.

    (A) and (B) are completely analogous and equally absurd.

  201. Bruce Smith Says:

    Robin Hanson #194: I skimmed your paper and consider it excellent.

    I want to suggest an enhanced model that I think your paper doesn’t cover. This model has GCs, and also a variant that you don’t mention — a civilization which *does* take control of events in a rapidly expanding volume (in particular, preventing other expanding civs from forming there), but which *is not* readily visible from far outside that volume. This falls outside your definition of GC, which requires a GC’s controlled volume to be “visibly different”; so I’ll call it “SEC” for “stealthy expanding civilization”. (Like you, I’ll assume we’re not already in either a GC- or SEC-controlled volume. I’ll also define an EC as either a GC or SEC, and assume GCs and SECs differ only in visibility, not in other ways relevant to your model. So your model applies unchanged to ECs, except only the GCs among them are visible.)

    Suppose the density ratio of SEC / GC is high. (This requires some strong assumptions about how a civ *could* evolve, but I think they are not impossible; and there are plausible strong motives for a civilization to try to adopt a “stealthy” strategy.)

    Then I think your model would apply unchanged to the sum of GC and SEC density, i.e. to EC density, except for one aspect — you would no longer have the observation of lack of close-enough-to-be-visible-if-they-werent-hiding ECs, but only the weaker one of lack of visible GCs.

    How would this change your conclusions?

  202. Gerard Says:

    TDW #198

    > But be aware that modern black contracting ensures the answer is unknowable.

    It’s certainly not supposed to work that way. All government contracts should be carried out under the supervision of some program office within the executive branch.

    I’ve heard claims that completely autonomous programs exist with no effective government oversight despite being financed by the government but I’ve seen no hard evidence for those claims and until I do I’m going to put them into the same category as all of the other conspiracy theories that are so prevalent in the ufology communities.

  203. red75prime Says:

    Scott #92:

    > the only such winning ticket that would ever arise anywhere in the universe, then how do you explain how anomalously early we’ve shown up in the universe’s history?

    Is there anything to explain? Let’s look at a distribution of potential civilizations. Those, who are anomalously early, will find themselves anomalously early. We found ourselves anomalously early, thus we belong to a class of civilizations who think that they aren’t exceptional and try to explain they early existence by the reasons other than statistical fluke.

    Of course, we need to act upon those other reasons for security considerations. But their validity can only be confirmed by positive observations.

  204. anonymous Says:

    The only practical way of expansion which I can imagine is if we freezed sperm and sent sperm + artificial wombs to distant planets, and only triggered the birth of the whole colony after the vehicle reached it’s destination. This could allow colonization of planets after hundreds of years of travel without needing to sustain bored human beings in a journey that takes their lifetime. I don’t think humans will ever leave the solar system alive, we’re not built for space travel.

  205. James Cross Says:

    #201 Bruce

    I thought of somewhat the same. I find this criteria somewhat problematic.

    c) clearly change the look of their volumes (relative to uncontrolled volumes)

    What sort of change would constitute a sufficient change for evidence of a GC? There seems to be an implicit bias in favor of vast engineering projects that wouldn’t violate physical laws but would show up as looking like something not natural.

    What exactly does “control of a volume” mean?

    This seem to imply dominance over other life forms and civilizations in the volume, perhaps to the end of extermination.

    Wow! Sounds a lot like humans, doesn’t it?

  206. Overcoming Bias : Experts Versus Elites Says:

    […] Twitter, in contrast to far more comments when arguable-elites discuss it in panelist/elite mode: Scott Aaronson (205), Scott Alexander (108), and Hacker News (110). People are far more interested in engaging […]

  207. Hanson, Robin Says:

    yenwoda #197, I wouldn’t claim exponential growth, as only quadratic growth can fit into a speed-limited space.

    J #199, our model does explain why humans are early.

    Bruce Smith #201, if GC might be invisible, then we can’t know we aren’t now in their volume of control, and we can’t use the fact we don’t see them to infer the speed they must travel at. At which point we can no longer estimate two of our three free parameters. Which puts us back near Drake equation territory.

    red75prime #203, by your argument there are never any suspiciously unlikely events. “Yes this person I hated and threatened to kill just seems to have died randomly; so what?”

  208. Bruce Smith Says:

    James Cross #205:

    > What exactly does “control of a volume” mean?

    It just means you can decide what kind of thing can happen there — not that you will necessarily decide that in an evil way!

    > This seems to imply dominance over other life forms and civilizations in the volume, perhaps to the end of extermination.

    It does mean “dominance” in the sense that, in that volume, we get to decide, and they don’t (except to whatever extent we allow that). But in principle, we could use that control reasonably, fairly, wisely, etc. It needn’t imply “extermination” and I hope it won’t. I think there are good arguments that it ought not to, and a typical EC will be intelligent enough to find and understand those arguments, so the remaining question is whether (considered as one unit) it will be sane enough to act on them.

  209. Bruce Smith Says:

    James Cross #205, addendum: Perhaps the word “govern” would be more appropriate than “control” in this discussion. I think it’s equivalent, in the context of what an EC does (by definition) to “its volume”, but suggests better analogies if you want to compare that to things humans do now.

  210. Gabriel Says:

    Do I understand correctly that Lev Gordeev is *not* claiming here that N != NP (even though the abstract contains the phrase “…and hence P is not NP”)? https://arxiv.org/abs/2005.00809 If so, can someone briefly explain to me in simple words what this paper *is* claiming?

  211. Scott Says:

    Gabriel #210: It explicitly claims to prove P≠NP in the last sentence of the abstract. I see that the same author previously proved NP=PSPACE. Given that the same results were proved by many, many previous arXiv preprints, I’d need a good reason to spend more time on this one! 🙂

  212. Raoul Ohio Says:

    Intelligent Life Derby:

    Consider two Intelligent Life Scenarios enjoying current consideration:

    SSS (the Robyn Hansen “Serious Spreading Sphere” theory).

    OOS (the Avi Loeb “Oumuamua is Obviously a Spacecraft” theory).

    Which do you find more likely?

    Cast your vote now!!

  213. Tamás V Says:

    I like this from Scott:

    For myself, I’d simply observe that trying to reason about matters far beyond current human experience, based on the microscopic shreds of fact available to us (e.g., about the earth’s spatial and temporal position within the universe), has led to some of our species’ embarrassing failures but also to some of its greatest triumphs.

    So here is my attempt (seriously): by the time a civilization becomes technologically capable to expand like hell, they are likely to also discover that there is a place (maybe another universe) that is much more pleasant to live in. So they just leave. Fermi paradox solved 🙂

    Unfortunately, I cannot turn this into a model, so cannot really reason about it either, therefore I don’t expect any triumph in this case.

  214. anthropic Says:

    I’d like to mention that I’ve formed my views on anthropic reasoning (as expressed in my previous comments) after reading an article by Radford M. Neal years ago:
    Puzzles of Anthropic Reasoning Resolved Using Full Non-indexical Conditioning
    Hanson’s argument is not compatibel with Neal’s approach.

  215. Bruce Smith Says:

    Robin Hanson #207:

    If I assume, more specifically, that most GCs are invisible, that we are not in any GC sphere of control (even though there is no visual evidence for that), and (for independent reasons than your argument) that their expansion speed is high, then am I correct in taking your model as an argument that apparently (1) we are early (relative to not assuming we’re not in a GC sphere of control), (2) there is a decent chance there are large-angle invisible spheres of control in our sky, unless the expansion speed is extremely close to c?

  216. red75prime Says:

    Hanson, Robin #207:

    > “Yes this person I hated and threatened to kill just seems to have died randomly; so what?”

    Not exactly “So what?”. It’s more like “As unlikely as it may sound, it could be a coincidence. Let’s investigate potential killer, while waiting for autopsy results.”

  217. Adam Brown Says:

    It’s interesting to compare two possible solutions to the Fermi Paradox:

    1. We don’t see them because Grabby Aliens expand at near light speed, annihilating civilizations in their path and therefore excising most of their future light cone.

    2. We don’t see them because advanced aliens tend to build colliders that trigger the decay of the Higgs vacuum (or a bubble of nothing, or…) and the resulting bubble of true vacuum expands at literally the speed of light, annihilating civilizations in their path and therefore excising most of their future light cone.

    These two theories have a lot in common! In the first there’s a social phase transition produced by advanced civilizations; in the second there’s a physical phase transition. Either way, it’s hard to survive in the future lightcone of the phase transition.

    It seems like all the anthropic/”why now?” arguments that would support theory 1 would equally support theory 2? And that theory 2 has the advantage that (i) you don’t need to come up with an explanation for why “you” were not born a grabby alien (the true vacuum is definitely not in your reference class!), and (ii) the lightspeed of the bubble wall is forced on you by the laws of physics, rather than something you need to derive.

  218. Phillip Dukes Says:

    “Grabby Aliens”? I’m left to imagine what that could possibly mean, up to and including individuals so self-centered they can’t cooperate. I have an even harder time with the defining criteria “never die alone”. Does this imply a population so dense that individuals are never out of sight of each other or is it supposed to mean these alien societies never self-destruct? But what is the justification in assuming this would be true for a society which is “Grabby” to any extent?

    THE AUTHORS ARE FLIPPANT!

  219. J Says:

    Adam #217
    3. You don’t even need to build colliders. Vacuum tends to decay so easily that most branchs of the universe soon end up dying. We’re only alive because of the many-worlds unsuicide trick, which also makes us both early and exponentially likely lonely.

  220. JimV Says:

    “We do NOT assume aliens can expand at near the speed of light. We instead assume they expand at some speed, and then INFER a fast speed from the fact that we don’t now see them.”–Dr. Hanson

    Yes, I got that from Dr. Scott’s outline here. As I understood it, that fast speed was supposed to be fast enough to outrun radio/EM transmissions and thus explain why we haven’t detected such transmissions, which would imply near light speed. Whether assumed or inferred, that answer is wrong according to physics as we know it. If it was inferred from a model, it means the model and/or inference is wrong. Isn’t that how science is supposed to work?

  221. Bruce Smith Says:

    Adam Brown #217: I think you’re right, and that’s the most depressing idea I’m likely to learn all day! I can only hope (without any real justification that I know of) that case (2) (suitably generalized) is a priori much less likely / densely occurring than case (1).

  222. Bruce Smith Says:

    JimV #220: As I understood it, that fast speed was supposed to be fast enough to outrun radio/EM transmissions …

    Not correct. An example from the paper (figure 9, if I’m interpreting it correctly): if the expansion speed is 50% of c, and there are n = 24 hard steps in GC formation, there’s still about a 12% chance a GC won’t see any others (of any size) by the time it starts expanding. This chance grows with speed/c, and reaches 1 when that ratio does. So for speed/c of 90% it’s much higher than 12% (though I can’t find the exact figure in the paper just now).

  223. Clint Says:

    So, as far as humans being the only evidence of a life form that survived the Drake equation … Isn’t being surprised to find oneself in a low probability outcome the definition of finding oneself in a low probability outcome?

  224. Bennett Standeven Says:

    @JimV:

    That was the impression I got from the outline, too. But after leafing through Hanson’s paper, it seems the fast speed just lowers the time-to-saturation enough that a typical exterior point won’t have many expansion regions in its past light cone. So an expansion at, say, 30% of light speed would still count as “fast”.

  225. HONEST ANNIE Says:

    Hanson’s model is an ecological model. It assumes that modelled entities don’t reflext the model and respond to it. With intelligent agents simple ecological modelling does not work. You need to use games theory and strategic thinking.

    Assuming intelligent aliens, they know this model.

    Assuming stable state alien civilizations, they have countermeasures.

    Grabby Aliens might discover immune system against cancer-like civilizations growing without limit. It should hit them early before they are metastasized.

  226. JimV Says:

    Thanks for the information. I am inclined to think 30% of c is way too fast, e.g., no way we could take off from the moon and reach 30% of c half-way to the nearest star (then have to decelerate), 5% would be about the limit. That’s based on calculations I did long ago, and maybe could be improved, but I would need to see the new data. At 5% of c I didn’t see the need to use the relativistic mass-increase, which would make the energy requirement greater and greater. Without a realistic calculation of space-travel speeds the model still seems naive to me.

    Vernor Vinge knew the science and based his interstellar empires on the fictional idea that the speed of light increases with distance from the center of mass of the galaxy due to unknown physics. My point is that he knew something like that was necessary, because according to science as we know it there will be no interstellar empires.

    My other point is that we have had engineers and scientists like Heinlein and Vinge dreaming of space travel in a big way since at least the 1950’s. Many of them put serious thought into it and made numbers, and concluded that unknown physics is needed, which has long been the consensus. Some of them worked at NASA as their day jobs and published papers on the limitations of space flight. Freeman Dyson put some thought into it also. Dr. Hansen has the right to take a hack at it, but his model seems about 70 years late to me.

    (90% of c is right out, as Monty Python would say.)

  227. JimV Says:

    That should be Dr. Hanson, sorry.

  228. asdf Says:

    At T-minus three frames, a sudden appearance of farmland and cities on the coasts those continents. At T-minus two frames, half the continent. At the second to last frame, a roaring interconnected network of roads, cities, farms, including skyscrapers in the cities that were just trying villas three frames ago. And in the last frame, nearly 80 percent of all wilderness converted to some kind of artifice, and the sky is streaked with the trails of flying machines all over the world.

    This is a plot element of the novel Dragon’s Egg, by Robert L. Forward. Humans discover life on the surface of a neutron star: there is a sentient species that lives a million times faster than humans, so their lifespan is around 40 minutes. They go from a primitive to a technological society before the very eyes of the human visitors watching from orbit. A little while later they have technologically surpassed humans.

    https://en.wikipedia.org/wiki/Dragon%27s_Egg

  229. anthropic Says:

    From Critiquing the Doomsday Argument by Robin Hanson (1998):
    “All else is not equal; we have good reasons for thinking we are not randomly selected humans from all who will ever live. You should include everything you know when doing inference, and you usually know things that imply you are not random. … Anyone who shows up when the population is well below the largest size that one’s prior gives stubstantial weight to knows they must be unusually early.”

    That’s right, and he should have sticked to that wisdom. If you expect a large number of civilizations near white dwarfs in the far away future but find your own near a sun-like star in the early days of the universe, then you have no good reason to think of yours as a typical sample of all civilizations of all time that have just become technological.
    Let’s accept Hanson’s parametrized model and let’s vary the expansion speed s keeping the other parameters fixed. He says that we should prefer the models with high s as they’d predict a higher percentage of all civilizations at the point of becoming grabby haven’t seen aliens. In fact we should prefer the models with low s because they predict a higher number of civilizations at our known time and stage of development that haven’t seen aliens, making our experience more likely.

  230. OhMyGoodness Says:

    As long as they stay away from our Lithium we should be fine.

    I tried to imagine why alien spacecraft would be here conducting surreptitious observations and concluded it must be for our stealth technology. Based on popular claims they are woefully behind Earth in stealth technology.

  231. Mitchell Porter Says:

    HONEST ANNIE #225… Wouldn’t such an immune system have to spread throughout space too? It would itself be a form of expanding technological civilization.

  232. Pascal Says:

    This is a question for Scott. If I understand correctly your take on Hanson’s paper, you find plausible the hypothesis that extraterrestrial civilization exist, and that some of them expand at nearly the speed of light. But in reply to my first comment, you also write:

    “Sorry to say, but my prior for extraterrestrial intelligences to travel all the way here, across hundreds of light-years, just to bounce around over the ocean in something that looks like a Tic-Tac and weird out some military pilot, is so astronomically low that even if this or that YouTube video were able to multiply the prior probability by a factor of 100, it still wouldn’t rise to the level where I’d be devoting any serious thought to it.”

    Aren’t you in a severe state of cognitive dissonance here? I mean, if some extraterrestrial civilizations expand at the speed of light, isn’t it plausible that some of their ships would end up on dear planet Earth?

  233. Scott Says:

    Pascal #232: The part that I find implausible is not the part where aliens expand at nearly the speed of light. Rather, it’s the part where they arrive here on earth only to dance around over the ocean in floating tic-tacs.

  234. Tim Gross Says:

    all of these Fermi discussions have no problem just assuming that the self-sample is accurate- that we are a species of ape on a terrestrial planet around a star at such and such time and place- but if we are doing an idealist probability analysis of our cosmic situation- we should NOT assume that interpretation our observations! we should assume that we are in a simulation- and that the aesthetic setting of the simulation tells us nothing about our cosmic state-

    being a simulation means we probably are a typical sample of intelligence- likely living in a scape generated by the degenerate matter computation surrounding ultramassive black holes during the googol years of the Black Hole Era- this is when virtually every observer exists- not literal matter based beings living during the brief trillion year afterflash of the Big Bang-

  235. Pascal Says:

    Scott #133: do you really believe that you have a sufficiently good prior to figure out by the sheer force of Bayesian reasoning what alien civilizations are supposed to do after their arrival on Earth? Like, do you think they should fly over Washington (*) instead of bouncing over the ocean?

    (*) Oops. They’ve already done that!
    https://en.wikipedia.org/wiki/1952_Washington,_D.C._UFO_incident

  236. Scott Says:

    Pascal #235: Well, almost all my probability mass is on stuff that’s either

    (1) totally undetectable to us, or
    (2) a complete and obvious game-changer for life on Earth.

    For some UFOs to float around, “detectable” only in grainy videos and scattered eyewitness reports, always below the threshold where it becomes an accepted fact, is just too much like what we would see if this were all, as the saying goes, the known irrational characteristics of terrrestrial intelligence rather than the unknown rational efforts of extraterrestrial intelligence.

  237. Gerard Says:

    Scott #236

    They could be performing experiments to measure the level of our collective intelligence/rationality by exposing us to stimuli that don’t fit our expected models of reality.

    I’m not saying that hypothesis is the most likely explanation for these phenomena but I think it deserves at least some weight.

  238. Robert Miles Says:

    I know this is not the point but it’s important: Please get a better mask! It’s easy to do and makes a big difference!

    https://www.microcovid.org/blog/masks

  239. Pascal Says:

    Scott #235 and Gerard #236: the two main explanations that I’ve found on the internet for such a sneaky behavior could be called the “nice alien theory” and the “bad alien theory”.

    In the first one, the aliens do not want to interfere too much with human civilization, perhaps out of some ethical concerns. They are afraid that coming into the open could cause widespread panic, perhaps even a collapse of human civilization as we know it (think of what happened in human history when 2 civilizations with very different degrees of technological development came into contact). It could also be called the “zoo theory”.

    In the second one, the aliens want to exploit some of the Earth’s ressources in ways that we do not fully understand, and perhaps take full control someday. But they do it in some stealth, sneaky way rather than all-out war.

  240. Jeff Says:

    When considering case 3: species serious about spreading widely, is there always a point where that spread would become counterproductive and the species would turn to either case 1 or 2? For a “grabby” species spreading out at near light speed, won’t they eventually reach some distance where communication/cultural mingling becomes impossible? Regions would evolve their own identity, and over time you’ve gone from expanding to protect yourself from other expanding species to having essentially seeded the galaxy with competitors at your same level of technology.

  241. Gerard Says:

    Pascal #239

    There’s no shortage of possible explanations. Maybe they’re keeping us under surveillance to determine if/when we are likely to become a threat to other civilizations. Probing our reactions to gauge our intellectual abilities, like I suggested, could be a component of that surveillance.

    The problem is it’s all just speculation with virtually no hard evidence and nothing that even ties the most credible UAP sightings to ET’s in any way.

    As you yourself pointed out speculating about the motives and strategies of literal aliens, who could well be millions of years more advanced than us, is unlikely to lead to accurate inferences.

  242. James Cross Says:

    Maybe this is how they have gotten here.

    “Pais continued to toot his own plasma horn. “Do realize that my work culminates in the enablement of the Pais Effect (original physical concept),” he said. “Such high energy [electromagnetic] radiation can locally interact with the Vacuum Energy State (VES) – the VES being the Fifth State of Matter (Fifth Essence – Quintessence), in other words the fundamental structure (foundational framework), from which Everything else (Spacetime included) in our Quantum Reality, emerges. The Engineering of the Pais Effect can give rise to the Enablement of Macroscopic Quantum Coherence, which if you have closely been following my work, you understand the importance of.”

    https://www.vice.com/en/article/4adpv9/us-navy-has-patents-on-tech-it-says-will-engineer-the-fabric-of-reality

    I did check the calendar to make sure it wasn’t April !.

  243. Lou Scheffer Says:

    Barkley #137, Scott #140, and Peter #150. All of you write of the Sun boiling away the ocean in a mere billion years. This is correct as we understand it, but *only if we don’t move the Earth further out*. Moving a planet seems hard but it something we can almost do today. Remember when commentators note that a spacecraft flyby also changes the orbit of the planet? Same with an asteroid close encounter – and one every few thousand years is plenty to keep pace with the expanding sun. And they are easy to arrange on a few thousand year notice – it just takes a tiny nudge. And you use the sensitivity of initial conditions to help even more – you nudge a small asteroid, so it comes close to a bigger asteroid, and so on. And you won’t run out of suitable asteroids – you can set up a single one to get deflected alternately by Jupiter and Earth, slowly moving Earth out and Jupiter slightly in. Peer-reviewed paper in a serious journal can be found here: https://arxiv.org/abs/astro-ph/0102126

  244. botnet-client Says:

    Maybe some proportions are coincidental, or maybe the relationship between the estimated age of the Earth and universe are a requirement for something outside of past consideration, or some epistemical issue.

    Obviously if there is an equal probability of any civilization being at an equivalent technological level at any point in time, that alone implies a Drake multiplier of a billionth, at minimum.

    Speculation that Tabby’s Star has a civilization nearing Kardashev type 2 is interesting only for arguing against some sort of Great Filter. Maybe the filter is incredibly simple, it requires frighteningly little intelligence to construct an atomic bomb (or a large pyramid for that matter) and engage in primitive behavior over it. Maybe the filter is literal.

    Or the Drake multiplier is about a quintillionth and we are the only sentient life in the Local Group.

  245. Bjørn Kjos-Hanssen Says:

    Thanks for the great blog.

    Minor suggestion: you you can change your favicon at https://scottaaronson.com/favicon.ico to something reflecting the topic of the blog… right now it’s the same favicon as my local wine store!

  246. Gerard Says:

    Re: botnet-client #244

    It looks like the singularity is still far.

  247. fred Says:

    Lou #243

    “Moving a planet seems hard but it something we can almost do today.”

    If we could, that would be an easy way to counter the effects of global warming? 😛

    But a quick back of the envelope calculation (I could be totally wrong):
    the change of energy required to extend the orbit of the earth by 1% is 125,000 billion times the energy of the Tsar Bomb, the largest nuclear weapon ever detonated, at 210 PJ.

    (the total energy of orbit is -GM1M2/2R)

  248. Observer effect Says:

    About why humans appeared so early, here is my (anthropic) argument:

    I divide universe into two parts: the life part and the non-life part. The life part is everything that contributed directly to life: Earth, solar system, whatever caused solar system to exist (to a lesser extent). Everything else is non-life part. There is no clear boundary though. Some things can be both.

    Anthropically, only the the life part of the universe need to exist. The non-life part doesn’t need to exist: it exists only as a bi product of the life-part. As a result, the non-life part of the universe will be minimized. Which is why we don’t find ourselves in a 100 billion year old universe.

  249. Lars Says:

    “sphere expanding at nearly the speed of light, transforming everything in its path”

    I don’t really know what that is supposed to mean, but I actually don’t need to to assess it’s plausibilit (which I would categorize as “nott !!!” — or maybe I left off a few !’s)

    Any expanding sphere — whether it be light or anything else — is subject to the inverse square diminution law. And whatever it’s makeup, such a “wave” that is (supposedly ) “transforming” everything in its path is going to be losing energy with distance travelled in addition to the inverse square diminution.

    Whatever the “effect”, the associated energy would necessarily diminish with distance until becoming utterly insignificant (if not undetectable) at some distance.

    Even if some highly advanced civilization were able to somehow harness the energy of black hole collisions to “power” their “expanding sphere” , they still could not avoid the diminution dictated by the inverse square law.

    After all, we know how small the gravitational wave effect can be on earth even for something with the enormous energy output of the collision of two massive black holes that collided billions of years ago.

  250. lars Says:

    Should have made clear that the diminution is in energy intensity (energy per unit area)

  251. Amoss Says:

    The model proposed seems to have the following assumptions:
    1. A random distribution of GCs
    2. A constant speed of expansion for each GC.

    There are many ways in which each of these assumptions could be violated (e.g. for 1. we know nothing of how starting position relates to the probability that a civilization makes it to the post-industrial GC phase, or for 2. we don’t know how complexity of the hardware substrate or the software running on it will induce feedback loops with the speed of expansion).

    But, ignoring these unknown unknowns and taking the model at face value: does it not just reduce to a weighted Voronoi diagram? e.g. the weight is a function of age and speed, and each point in the partitioned space is closest by weight to a particular GC starting point.

  252. anthropic Says:

    Hanson, Robin #194: “I get that it can be fun to engage exotic anthropic arguments, but our argument doesn’t depend on them.”

    Not true. The inference that the expansion speed is fast depends on the assumption that we are typical among all civilizations at our state of development. This kind of anthropic argument has been discussed and criticized by several authors, e. g. James B. Hartle, Mark Srednicki: Are We Typical?
    (Phys.Rev.D75:123523,2007)
    Let me quote some of their conclusions:
    • A theory is not incorrect merely because it predicts that we are atypical.
    • Theories are tested using our data as a collectivity of human observers. What other observers might see, how many of them there are, and what properties they do or do not share with us are irrelevant for this process. …
    • We have data that we exist in the universe, but we have no evidence that we have been selected by some random process. We should not calculate as though we were.

  253. Mark Srednicki Says:

    I agree with commenter anthropic, #229: I believe that Hanson was right in 1998, and wrong now.

    Scott’s question “how do you explain how anomalously early we’ve shown up in the universe’s history?” seems to me to be akin to “how do you explain that we are not insects, given that humans are vastly outnumbered by insects?”.

  254. Mark Srednicki Says:

    AND I just noticed that anthropic actually cited me in the post just before mine, so no surprise that we agree. (Moral: read to the end of the comments before commenting.)

    Executive summary of my papers on this with Jim Hartle: typicality assumptions are dangerous, and should therefore always be carefully spelled out.

  255. Peter Erwin Says:

    Lou Scheffer @ 243

    The point of the argument isn’t actually “how long into the future could a technological civilization survive on Earth?” — it’s how long would the Earth remain habitable in the absence of technological interventions? In other words: what’s the upper limit on the possible time for life (or multicellular life, or actual intelligence) to appear on something like the Earth? The answer seems to be “shorter than you’d think, if you just went by the main sequence lifetime of the Sun”.

  256. Lou Scheffer Says:

    Robert #103, Greg #105, and Scott #118: I find the Sandberg, Drexler, & Ord paper unconvincing. The final result N does indeed depend on the form of the distribution used – that I agree with. But what they do next is replace a distribution they do not like with one they like better. The problem arises because we don’t know the shape of the distribution either, and this has a huge effect. They pick log-uniform for 6 of the 7 parameters, but without an understanding of the mechanism this is just a guess, and one biased towards small results.

    Scott #176: Breakthough listen, and other existing radio telescope technology, could not detect another Earth unless they got very, very lucky. They would need to be directly in the path of some highly beamed, high power transmission, which would need to be repeated otherwise it could not be confirmed (and no-one will believe it without confirmation). The largest of our (more or less) omni-directional signals (radars, analog TV stations, etc) are only detectable by our largest radio telescopes at a little less than a light year distance.
    However, aliens with telescope technologies only a century or so in advance of ours could decode our TV signals from any distance they have reached. See https://www.lscheffer.com/tv.pdf for an analysis.

    Fred 247: We can’t use the planet moving techniques we know of now to help with global warming since the time scale is just too slow. The first step, for example, is tweak some asteroid orbit now in such a way that it flies by the Earth sometime in the next 6000 years, and moves the orbit out by 0.001%. This kind of rate is fine for dealing with billion year time constants, but global warming needs action on decade time scales.

  257. Tom Grey Says:

    I think a version of the Dark Forest is likely, and sad – look at how much humans enjoy HATING different humans because … reasons. (I’m thinking Dems who hate Trump & now Trump-supporters).

    Recently I’ve been thinking about the Moon, again.
    It might be that the extra instability of sun + moon ocean tides accelerated key evolution changes by some percentage, like 20-40%. So such “big-moon” blessed water planets like Earth both have faster evolution AND are far far rarer than similar planets with much smaller moons.
    [This was the first post of yours I read. Then your earlier one which I’ll comment on now, hopefully spelling excellent correctly.]

  258. John Baez Says:

    I’m glad Robin Hanson is now recognizing the work of S. Jay Olson on aggressively expanding extraterrestrial civilizations:

    * Robin Hanson, Hail S. Jay Olson, Overcoming Bias, February 21, 2021.

    Jay came to the physics department of U. C. Riverside to work with me on quantum gravity. We published a paper about fundamental limitations on the measurement of lengths due to quantum gravity effects. He failed his quals—I think thermodynamics was the killer—but then bounced back, getting his Ph.D. at Louisiana State University, which had a good team working on loop quantum gravity. He contacted me after he wrote three papers on aggressively expanding extraterrestrial civilizations in 2015, and I blogged about them in February 2016.

  259. Pascal Says:

    A belated complement to Gerard #202:
    “I’ve heard claims that completely autonomous programs exist with no effective government oversight despite being financed by the government but I’ve seen no hard evidence for those claims and until I do I’m going to put them into the same category as all of the other conspiracy theories that are so prevalent in the ufology communities.”

    I just stumbled on “an information site on the UFO phenomenon by and for professional scientists.” At https://www.ufoskeptic.org/president/ they explain how secret programs could operate without even the President’s knowledge. I have of course no means of determining if this is reliable information, or just another conspiracy theory…

  260. Pascal Says:

    At the risk of looking like a conspiracy nutcase, I’ll add the following quote:

    “However, some former USS Princeton crew members have openly stated that the Aegis radar data collected during the incidents in November 2004 was soon removed from the ship by individuals who arrived aboard the Princeton by helicopter shortly after the “Tic Tac” encounters occurred. That critical radar data still appears to be missing. Moreover, the deck logs of the USS Princeton for November 2004 are also strangely and inexplicably missing from the National Archives, raising the extraordinary possibility that some secretive USG component is working to conceal UAP incidents and information. ”

    Who do you think is the crazed author of this remarkable statement? Incredibly, this is none other than Christopher Mellon, former Deputy Assistant Secretary of Defense at the Pentagon. Source: https://dpo.tothestarsacademy.com/blog/potential-sources-of-information-regarding-unidentified-aerial-phenomenon

  261. anthropic Says:

    Mark Srednicki #253f.: Thank you for your support. (And you didn’t just agree with yourself as I read your paper in the time between my last two comments.)

    John Baez #258: From Olson’s comment as quoted in Hanson’s post you linked to:

    “Robin has a knack for forcing the issue. If their “hard steps model” for the appearance rate of life is valid (giving f(t) ~ t^n), there aren’t too many ways to solve humanity’s earliness problem. Something would need to make the universe a very different place in the near cosmic future, as far as life is concerned. A phase transition resulting in the “end of the universe” would do it — bad news indeed. But the alternative is that we are, literally, the phase transition.”

    That’s obviously very similar to the Doomsday argument, which makes basically the same point about our generation’s place in the timeline of human history. Supposedly, imminent doom is needed to explain our (apparent) earliness. It’s a strange argument and Hanson used to reject it (see #229). I don’t understand why so many people seem to believe that it’s valid, or at least the grabbing aliens version thereof.

  262. Elan Stopnitzky Says:

    Isn’t the whole point of having a civilization to convert free energy into heat? In which case a sphere seems like a bad design, because the power you can radiate scales with your surface area as R^2 and the power you’d like to consume scales with your volume as R^3. So an expanding spherical civilization would have to sink greater and greater shares of its energy budget into cooling if it wants to maintain the same power density and temperature, which at some point negates the additional power it can harness by expanding its reaches. A quick calculation shows that if it’s dissipating heat through the Stefan-Boltzmann law, then the maximum power density it can maintain is proportional to T^4 / R, where T is its temperature and R is the radius of the sphere. And if the civilizations don’t expand in spherical shells then that significantly changes the picture because, e.g., a lengthening cylindrical civilization doesn’t gobble everything in its line of sight.

  263. botnet client Says:

    @elan : Deconstructing the Earth to create a Dyson sphere or other megastructure is an interesting challenge. If fracking earthquakes or mine pillar explosions are any indicator, the Earth will rupture at the seams while mining operations are underway. Not certain if it will be from a release of pressure or from a reduction of temperature leading to a reduction of pressure.

    As to your argument about heat distribution, directional radiators exist. If civilizations’ purpose is to convert energy, than it wouldn’t have to be done efficiently in terms of work done, only in waste heat removed.

  264. David Piepgrass Says:

    I can never quite get over my astounding luck at being alive at this point in history (or my failure to usefully capitalize on it.)

    Life is a negligible portion of all matter, yet I am alive. Quadrillions — quintillions? — of animals came before me, and yet I am none of them; instead I am me. Even among the humans, I was grateful, growing up, to live in the time of my favorite things in the world, the computers. Somehow I did not live 100 years ago when there were no computers, or 1000 years ago, or 10,000 years go, or 100,000 years ago, or 1,000,000 years ago. Even among the humans of the 21st century, maybe only a million or two have noticed how incredible the situation in which they find themselves is. And yet I am one of them. Somehow I am one of the most unique people alive, and I live at what seems like a perfect time for a person like me. The anthropic principle just doesn’t alone do justice to the amazingness of my situation. And sometimes, amid wondering if it’s possible to earn enough money to achieve even modest things in this world, I step back and wonder how I can be alive at such a shockingly special place in the universe, at such a shockingly special point in history, in such a shockingly unusual being.

    At least I’m not the king of Saudi Arabia or something.

    A coincidence that incredible could really go to my head.

    > “if we want human-originated sentience to spread across the universe, then the sooner we get started the better!”

    “Sooner”, as in, within 1 million rather than 100 million years? It seems highly unlikely that it matters much whether we colonize the galaxy 5.543 or 5.544 billion years after Earth was formed. By far the most likely way we’ll spread “human” influence across the galaxy is via AGIs, because they will have a dramatically easier time making the journey. For that we need to figure out alignment, and somehow be sure we’ve got it right. These things may take lifetimes — at once an extreme amount of time, and no time at all.

    Good call noticing how early this planet lies in the history of the universe. It is said the universe was once much less hospitable to life, not just because of all the violent collisions, but also due to the rarity of more interesting elements out of which rocky worlds could form.

  265. Grabby Aliens – READING LIST Says:

    […] Shtetl-Optimized2021/01/30: Reddit2021/01/31: Hacker News2021/02/03: Marginal […]